Neurology Flashcards

1
Q

A 24-year-old woman presents to you, her GP saying that she has been feeling low in mood and tired over the last month. You want to assess her for possible depression. What first two questions do you ask?

A

NICE recommendation - yes to one of two questions have high specificity for depression

  1. During the last month, have you often been feeling down, depressed or hopeless?
  2. During the last month, have you often been bothered by having little interest or pleasure in doing things?
How well did you know this?
1
Not at all
2
3
4
5
Perfectly
2
Q

24-year-old patient with possible depression: your patient answers yes to both your screening questions, what questions would you follow up with?

A

During the last month, have you often been bothered by:

  1. Feeling bad about yourself or that you are a failure or have let yourself or your family down?
  2. Poor concentration?
  3. Tiredness/low energy levels?
  4. Changes in appetite (reduced or increased)?
  5. Changes in your sleep pattern (sleeping too much, problems getting to sleep, waking in the night or waking early)?
  6. Being so slowed down, or so restless or fidgety, that other people have noticed?
  7. Thoughts of death?
How well did you know this?
1
Not at all
2
3
4
5
Perfectly
3
Q

What tools are used to assess patients newly diagnosed with depression?

A
  • Patient Health Questionnaire (PHQ-9)
  • Hospital Anxiety and Depression (HAD) Scale
How well did you know this?
1
Not at all
2
3
4
5
Perfectly
4
Q

What are the different classes of antidepressants?

A
  • Selective Serotonin Reuptake inhibitors (SSRI) e.g. sertraline
  • Tricyclic antidepressants (TCA) e.g. lofepramine or amitriptyline
  • Monoamine Oxidase Inhibitors (MAOI) e.g. selegiline.

Newer classes:

  • Serotonin Noradrenaline Reuptake inhibitors (SNRIs) e.g. duloxetine or venlafaxine.
How well did you know this?
1
Not at all
2
3
4
5
Perfectly
5
Q

What is St John’s wort?

A

OTC herbal remedy often used for depression

How well did you know this?
1
Not at all
2
3
4
5
Perfectly
6
Q

Why is it important to know if a patient is taking St John’s Wort?

A

It can interact with many other meds, it is a potent inducer of cytochrome P450 and other enzymes involved in drug metabolism.

This means prescribed medications may be metabolised differently and become more or less potent when taken alongside St John’s wort.

How well did you know this?
1
Not at all
2
3
4
5
Perfectly
7
Q

What medications does St John’s with interact with and make more potent?

A

SSRI antidepressant, increasing the risk of developing serotonin syndrome.

How well did you know this?
1
Not at all
2
3
4
5
Perfectly
8
Q

What medications does St John’s wort interact with and make less potent?

A

Combined oral contraceptive pill, warfarin, statins, digoxin, anticonvulsants and HIV medications (not exhaustive)

How well did you know this?
1
Not at all
2
3
4
5
Perfectly
9
Q

What is the pharmacology (Mech of action, side effects) of SSRIs?

A

Mech of action: Inhibit serotonin reuptake, increasing its concentration in the synaptic cleft.

Side effects:

  • Feeling agitated, shaky or anxious
  • Feeling or being sick
  • Diarrhoea or constipation
  • Reduced libido
  • Loss of appetite and weight loss
  • Difficulty achieving orgasm during sex or masturbation
  • Erectile dysfunction

Boxed warning: suicidal thinking in children, adolescents, and young adults

How well did you know this?
1
Not at all
2
3
4
5
Perfectly
10
Q

What is the pharmacology (Mech of action, side effects) of SNRI (Serotonin-norepinephrine reuptake inhibitors)?

A

Mech of action:
Inhibit serotonin and norepinephrine reuptake, increasing their concentration in the synaptic cleft

Side effects same as SSRIs plus:

Hypertension, tachycardia, closed-angle glaucoma, urinary retention

Boxed warning: suicidal thinking in children, adolescents, and young adults

How well did you know this?
1
Not at all
2
3
4
5
Perfectly
11
Q

What is the pharmacology (MOA, side effects) of TCAs (Tricyclic antidepressants)?

A

MOA
Inhibition of serotonin and norepinephrine reuptake into nerve endings

Side effects

  • Tachycardia
  • Prolonged QT intervation
  • Aggression
  • Dry mouth, reduced appetite
  • Serotonin syndrome

Black box warning: suicidal thinking in children, adolescents, and young adults

How well did you know this?
1
Not at all
2
3
4
5
Perfectly
12
Q

What is St John’s Wort’s pharmacology (Mech of action, side effects)?

A

It acts as a serotonin reuptake inhibitor. Reduce the uptake of serotonin at neuronal synapses, as well as dopamine and norepinephrine

Side effects:

  • Feeling nauseous or being sick
  • Diarrhoea
  • Headaches
  • Allergic reactions
  • Tiredness
  • Dizziness
  • Confusion
  • Dry mouth
  • Increased sensitivity to sunlight.
How well did you know this?
1
Not at all
2
3
4
5
Perfectly
13
Q

When are TCAs typically used?

A

Second-line treatment for major depressive disorder when there is no response to other antidepressants, such as SSRIs.

How well did you know this?
1
Not at all
2
3
4
5
Perfectly
14
Q

Define stroke

A

Stroke describes a neurological deficit lasting longer than 24 hours due to vascular compromise. The aetiology can be divided into ischaemic and haemorrhagic.

How well did you know this?
1
Not at all
2
3
4
5
Perfectly
15
Q

In a stroke/TIA, are emboli or thrombi aetiologically more important?

A

Emboli

How well did you know this?
1
Not at all
2
3
4
5
Perfectly
16
Q

What are the different types of strokes?

A
  • Ischaemic (85%)
  • Haemorrhagic (15%)
  • Transient Ischaemic Attack
How well did you know this?
1
Not at all
2
3
4
5
Perfectly
17
Q

What are the different classifications of stroke?

A

The most common is the Bamford classification:

  1. Total anterior circulation stroke (TACS)
  2. Partial anterior circulation stroke (PACS)
  3. Lacunar stroke
  4. Posterior circulation stroke
How well did you know this?
1
Not at all
2
3
4
5
Perfectly
18
Q

Bamford classification: define total anterior circulation stroke (TACS)

A

Total anterior circulation stroke (TACS) - 15% cases

Anterior or middle cerebral artery

All three of the following:

Hemiplegia
Homonymous hemianopia
Higher cortical dysfunction, such as dysphasia or neglect

How well did you know this?
1
Not at all
2
3
4
5
Perfectly
19
Q

Bamford classification: define partial anterior circulation stroke (PACS)

A

Partial anterior circulation stroke (PACS) - 40 - 45% cases

Anterior or middle cerebral artery

Two of three from:

Hemiplegia
Homonymous hemianopia
Higher cortical dysfunction, such as dysphasia or neglect

How well did you know this?
1
Not at all
2
3
4
5
Perfectly
20
Q

Bamford classification: define lacunar stroke

A

Lacunar stroke - 20%

Blood vessels affected:
Perforating arteries: usually affects the posterior limb of the internal capsule

Criteria:
There is no higher cortical dysfunction or visual field abnormality

One of the following:

Pure hemimotor (hemiplagia) or hemisensory loss (paraesthesia or numbness)
Pure sensorimotor loss
Ataxic hemiparesis (weakness on one side of body)

How well did you know this?
1
Not at all
2
3
4
5
Perfectly
21
Q

Bamford classification: define posterior circulation stroke

A

Posterior circulation stroke - 20% cases

Blood vessels affected
Posterior cerebral or vertebrobasilar (vertebral arteries merge ad give rise to the basilar artery) or branches

Criteria
One of the following:

Cerebellar syndrome
Isolated homonymous hemianopia
Loss of consciousness

How well did you know this?
1
Not at all
2
3
4
5
Perfectly
22
Q

What are the risk factors for developing ischaemic stroke?

A
  • Hypertension: Biggest risk factor
  • Age: the average age for a stroke is 68 to 75 years old
  • Smoking
  • Diabetes
  • Hypercholesterolaemia
  • Atrial fibrillation
  • Family history
  • Haematological disease: such as polycythaemia

Medication: HRT or the combined oral contraceptive pill

  • Cardiovascular disease such as angina, myocardial infarction and peripheral vascular disease
  • Previous stroke or TIA
How well did you know this?
1
Not at all
2
3
4
5
Perfectly
23
Q

What is an ischaemic stroke?

A

Reduction in cerebral blood flow due to arterial occlusion or stenosis.

Typically divided into thrombotic, embolic, and lacunar.

How well did you know this?
1
Not at all
2
3
4
5
Perfectly
24
Q

What is a haemorrhagic stroke?

A

Ruptured blood vessel leading to reduced blood flow.

Typically divided into:

  • Intracerebral: bleeding within the brain parenchyma
  • Subarachnoid: bleeding into the subarachnoid space
  • Intraventricular: bleeding within the ventricles; prematurity is a very strong risk factor in infants
How well did you know this?
1
Not at all
2
3
4
5
Perfectly
25
Q

What is the aetiology of ischaemic stroke?

A

Cardiac:

  • Atherosclerotic disease: smoking, hypertension, diabetes, high cholesterol
  • Atrial fibrillation

Vascular:

  • Aortic dissection
  • Vasculitides

Haematological:

  • Hypercoagulability e.g. antiphospholipid syndrome
  • Sickle cell disease
How well did you know this?
1
Not at all
2
3
4
5
Perfectly
26
Q

What is the aetiology of haemorrhagic strokes?

A

Intracerebral

  • Trauma
  • Hypertension

Subarachnoid

  • Berry aneurysm

Extradural and subdural haemorrhages are not considered haemorrhagic strokes.

How well did you know this?
1
Not at all
2
3
4
5
Perfectly
27
Q

What general signs and symptoms might a patient with a stroke present with?

A

In neurology, suspect a vascular cause when there is a sudden onset of neurological symptoms.

Stoke symptoms are typically asymmetrical:

Sudden weakness of limbs
Sudden facial weakness
Sudden onset dysphasia (speech disturbance)
Sudden onset visual or sensory loss

How well did you know this?
1
Not at all
2
3
4
5
Perfectly
28
Q

What is the clinical presentation of a stroke affecting the anterior cerebral artery?

A

Contralateral hemiparesis (weakness of one side of the body) and sensory loss with lower limbs > upper limbs

How well did you know this?
1
Not at all
2
3
4
5
Perfectly
29
Q

What is the clinical presentation of a stroke affecting the middle cerebral artery?

A
  • Contralateral hemiparesis and sensory loss with upper limbs > lower limbs
  • Homonymous hemianopia
  • Aphasia: if affecting the ‘dominant’ hemisphere (the left in 95% of right-handed people)
  • Hemineglect syndrome: if affecting the ‘non-dominant’ hemisphere - unaware and ignoring non-dominant side
How well did you know this?
1
Not at all
2
3
4
5
Perfectly
30
Q

What is the clinical presentation of a stroke affecting the posterior cerebral artery?

A

Posterior cerebral artery

  • Contralateral homonymous hemianopia with macular sparing (due to dual blood supply from MCA and PCA to macula)
  • Visual agnosia (inability to recognize objects, persons, sounds, shapes, or smells)
How well did you know this?
1
Not at all
2
3
4
5
Perfectly
31
Q

What is the clinical presentation of a stroke affecting the vertebrobasilar artery?

A

1) Cerebellar signs

  • Dysdiadochokinesia
  • Ataxia (gait and posture)
  • Nystagmus
  • Intention tremor
  • Slurred speech
  • Hypotonia/heel-shin test

2) Reduced consciousness

3) Quadriplegia or hemiplegia

How well did you know this?
1
Not at all
2
3
4
5
Perfectly
32
Q

What is the clinical presentation of a stroke affecting the retinal/ophthalmic artery?

A

Amaurosis fugax - temporary loss of vision in one or both eyes due to loss of blood flow to the retina

How well did you know this?
1
Not at all
2
3
4
5
Perfectly
33
Q

What is the clinical presentation of a stroke affecting the basilar artery?

A

“Locked-in” syndrome

How well did you know this?
1
Not at all
2
3
4
5
Perfectly
34
Q

What is Weber’s syndrome?

A

Midbrain infarct - affecting branches of the PCA.

Clinical presentation:

  • Oculomotor palsy and contralateral hemiplegia
How well did you know this?
1
Not at all
2
3
4
5
Perfectly
35
Q

What investigations and tests are used to diagnose stroke?

A
  • 1st line: non-contrast CT head: usually normal in the first few hours of ischaemic stroke but hyperdense blood if haemorrhage
  • ECG: assess for atrial fibrillation
  • Bloods: screens Hba1c, lipids, clotting screens. Helps exclude mimics such as hypoglycemia and hyponatraemia
How well did you know this?
1
Not at all
2
3
4
5
Perfectly
36
Q

What is the management plan for an ischaemic stroke?

A

Conservative: maintain glucose level, hydration and temp

Antiplatelet:

  • Aspirin 300 mg: ASAP after haemorrhage is excluded on CT
  • If thrombolysis, start aspirin 24 hours after once haemorrhage is excluded

Thrombolysis (meds to dissolve clot):

  • If < 4.5 hours of symptom onset (less effective after window) and haemorrhage excluded on imaging

Thrombectomy (surgery to remove clot):

  • Confirmation of stroke using CTA (CT of blood vessels) or MR angiogram before thrombectomy

Anticoagulation:

  • If AF is the cause - anticoagulation should only be started >14 days post-stroke. Aspirin 300 mg until then.
How well did you know this?
1
Not at all
2
3
4
5
Perfectly
37
Q

What are the secondary prevention strategies for ischaemic stroke?

A
  • 1st line: clopidogrel 75 mg daily lifelong is (after two weeks of aspirin)
  • High-dose statin: e.g. atorvastatin 20-80mg, usually after 48 hours of the stroke
  • Manage hypertension, diabetes, smoking and other cardiovascular risk factors
How well did you know this?
1
Not at all
2
3
4
5
Perfectly
38
Q

What is the management plan for a haemorrhagic stroke?

A

Depends on the subtype.

  • Subarachnoid haemorrhage usually requires endovascular or surgical clipping.
  • Admit to neurocritical care: intensive monitoring due to the risk of raised intracranial pressure and airway compromise.
  • ICP: consider intubation with hyperventilation, head elevation (30°) and IV hypertonic saline/mannitol.
  • Surgical intervention: decompression may be needed - treats compressed nerves
How well did you know this?
1
Not at all
2
3
4
5
Perfectly
39
Q

What medications are used in the secondary prevention of haemorrhagic strokes?

A

Ramipril - beta-blocker
Atorvastatin - statin

How well did you know this?
1
Not at all
2
3
4
5
Perfectly
40
Q

Define transient ischaemic attack (TIA) in a “time-based” definition

A

Transient ischaemic attack (TIA) refers to a period of transient cerebral ischemia resulting in a self-resolving neurological deficit within 24 hours.

Stroke features > 24 hours

How well did you know this?
1
Not at all
2
3
4
5
Perfectly
41
Q

Define TIA in a “tissue-based” definition

A

Definition of TIA moving from time-based to tissue-based:

A transient episode of neurological dysfunction caused by focal brain, spinal cord, or retinal ischaemia, without acute infarction.

How well did you know this?
1
Not at all
2
3
4
5
Perfectly
42
Q

What is the aetiology of TIA?

A

Similar to the aetiology of ischaemic stroke, usually caused by thromboembolic disease

How well did you know this?
1
Not at all
2
3
4
5
Perfectly
43
Q

What are the risk factors for developing TIA?

A

~ 15% of patients have at least one TIA before a stroke.

  • Increasing age
  • Hypertension
  • Smoking
  • Diabetes
  • Hypercholesterolaemia
  • Atrial fibrillation
  • Carotid stenosis
How well did you know this?
1
Not at all
2
3
4
5
Perfectly
44
Q

What signs and symptoms might a patient with TIA present with?

A

Symptoms

  • Facial weakness
  • Limb weakness
  • Slurred speech
  • Amaurosis fugax: a painless temporary loss of vision, usually in one eye

Signs:

  • Focal neurology on examination (signs specific to affected brain area)
  • Irregular pulse: suggests atrial fibrillation as an underlying cause
  • Carotid bruit (heart sound due to turbulent, non-laminar flow): suggests carotid artery stenosis
How well did you know this?
1
Not at all
2
3
4
5
Perfectly
45
Q

What investigations and tests are used to diagnose TIA?

A

ECG: rule out atrial fibrillation as an underlying cause

Bloods: screen for Hba1c, lipids, clotting screen and rule out hypoglycemia and hyponatraemia (mimics TIA)

TIA clinic: most recent guidance states that all patients should be seen within 24 hours of symptom onset

At the TIA clinic:

  • Specialist assessment
  • MRI head
  • Carotid doppler (USS - images of big arteries in the neck that supply the brain).
How well did you know this?
1
Not at all
2
3
4
5
Perfectly
46
Q

What is the initial management plan for TIA?

A

Acute:

1st line: antiplatelet: initially aspirin 300mg - until the patient is reviewed in TIA clinic.

Do not offer aspirin if:
Bleeding disorder or taking an anticoagulant: needs immediate admission and assessment

  • Carotid endarterectomy (removal of fatty plaques from carotid arteries): stenosis of > 70% on Doppler = urgent endarterectomy
How well did you know this?
1
Not at all
2
3
4
5
Perfectly
47
Q

What is the secondary prevention of TIA?

A
  1. 1st line: Clopidogrel 75 mg daily once the patient has been reviewed in TIA clinic and the diagnosis confirmed
  2. High-dose statin: all patients e.g. atorvastatin 20-80mg
  3. Manage hypertension, diabetes, smoking and other cardiovascular risk factors
How well did you know this?
1
Not at all
2
3
4
5
Perfectly
48
Q

Define intracerebral haemorrhage (ICH)

A
  • Type of haemorrhagic stroke, and thought to be caused by damage to small cerebral vessels, which results in bleeding into the brain tissue.
How well did you know this?
1
Not at all
2
3
4
5
Perfectly
49
Q

What are the aetiology/risk factors for intracerebral haemorrhage (ICH)?

A
  • Hypertension (most common)
  • Older age
  • Male sex
  • Asian, black and/or Hispanic
  • Illicit drug use e.g. cocaine
  • FHx of ICH
How well did you know this?
1
Not at all
2
3
4
5
Perfectly
50
Q

What symptoms and signs might a patient with ICH present with?

A
  • Unilateral weakness or paralysis in the face, arm, or leg
  • Sensory loss (numbness)
  • Dysphasia
  • Dysarthria (inability to control speech muscles)
  • Visual disturbance
  • Photophobia
  • Headache
  • Ataxia (impaired movement coordination)
How well did you know this?
1
Not at all
2
3
4
5
Perfectly
51
Q

What investigations/tests are used to diagnose ICH?

A
  • Non-contrast CT head - hyperdense area due to bleed
  • Serum glucose to rule out hypo and hyperglycemia as mimic
  • Serum electrolytes to rule out hyponatremia as cause for neurological symptoms
  • Serum urea and creatinine to rule out renal failure as contraindicated in some stroke treatments
  • Liver function tests to rule out liver dysfunction as a cause of haemorrhage
  • FBC to rule out thrombocytopenia as the cause of haemorrhage
  • Clotting screen to rule out coagulopathy as the cause of haemorrhage
  • ECG - abnormal ECG can indicate myocardial injury
How well did you know this?
1
Not at all
2
3
4
5
Perfectly
52
Q

What is the management plan for ICH?

A

Suspected ICH:

  • 1st line: Stabilisation using ABCDE, then refer to hyper-acute or acute stroke ward

Confirmed ICH:

  • 1st line: supportive care and monitoring:
  • Assess GCS
  • Maintain SpO2 > 96% (>88% if at risk of hypercapnic resp failure)
  • Maintain stable BP + blood glucose
  • Neurosurgery referral assessment if suitable
How well did you know this?
1
Not at all
2
3
4
5
Perfectly
53
Q

Define subarachnoid haemorrhage (SAH)

A

Subarachnoid haemorrhage (SAH) is a type of haemorrhagic stroke, characterised by blood within the subarachnoid space. It is most commonly caused by trauma (traumatic SAH).

Characteristic “thunderclap headache” - severe, sudden onset headache

How well did you know this?
1
Not at all
2
3
4
5
Perfectly
54
Q

What is the aetiology of SAH?

A

Most common - trauma (traumatic SAH)

Most non-traumatic cause - rupture of an intracranial aneurysm - accounts for 80% of cases:

Berry aneurysm - arises at arterial bifurcation of the circle of Willis, most commonly in junction between anterior communicating and anterior cerebral arteries

How well did you know this?
1
Not at all
2
3
4
5
Perfectly
55
Q

What signs and symptoms might a patient with SAH present with?

A

Symptoms:

  • Headache - Severe, sudden onset
  • Occipital
  • ‘Thunderclap’ headache
  • Meningism: photophobia and neck stiffness
  • Nausea and vomiting
  • Confusion, coma and seizures

Signs:

  • 3rd nerve palsy (oculomotor) - an aneurysm arising from the posterior communicating artery will press on the 3rd nerve, causing a palsy with a fixed dilated pupil.
  • 6th nerve palsy (abducens) - a non-specific sign which indicates raised intracranial pressure
  • Reduced GCS
How well did you know this?
1
Not at all
2
3
4
5
Perfectly
56
Q

What investigations/tests are used to diagnose subarachnoid haemorrhages

A

The investigations below should be performed alongside basic blood tests, including serum glucose (hyperglycemia in 1/3 patients) and clotting screen (elevated INR + prolonged PTT).

  • Urgent non-contrast CT head: diagnostic
  • CT imaging typically shows blood in the basal cisterns
  • ECG: should be requested for all patients and may demonstrate arrhythmias, ischaemic and ST-elevation (can mimic acute coronary syndrome/STEMI)
How well did you know this?
1
Not at all
2
3
4
5
Perfectly
57
Q

What is the management plan for SAH?

A
  1. Once SAH confirmed, refer immediately to neurosurgery
  2. Intervention should be performed within 24 hours due to the risk of rebleeding.
  3. 1st line: nimodipine give immediately upon diagnosis, prevents vasospasm - 21-day course
  4. Intervention:
    1st line is endovascular coiling of the aneurysm
  5. If features of raised intracranial pressure: consider intubation with hyperventilation, head elevation (30°) and IV mannitol
  6. Conservative:
  • Bed rest
  • Antitussive agent (anti-cold meds) and stool softeners: prevents straining and therefore reduce the risk of rebleeding
How well did you know this?
1
Not at all
2
3
4
5
Perfectly
58
Q

Define extradural haemorrhage

A

An extradural haemorrhage (also known as an ‘epidural’ haemorrhage) is bleeding into the potential space between the skull and the dura mater.

The blood then collects in this space and is referred to as an extradural haematoma (EDH).

Not a stroke!

How well did you know this?
1
Not at all
2
3
4
5
Perfectly
59
Q

What is the aetiology of extradural haemorrhage?

A

EDH is usually caused by trauma, e.g. blunt trauma to the head:

Arterial bleeding: the majority of EDHs occur as a result of rupture of the middle meningeal artery.

How well did you know this?
1
Not at all
2
3
4
5
Perfectly
60
Q

What signs and symptoms might a patient with extradural haemorrhage present with?

A

The exact presentation varies based on injury location, and most patients have skull fractures:

  1. Initial loss of consciousness
  2. Lucid interval in about 20% patients, regains full consciousness due to venous shunting of blood
  3. Worsening neurological status:
  • Local compression due to haematoma expansion = brain herniation
  • Reduced GCS
  • Cushing’s reflex:hypertension, irregular breathing and bradycardia
  • Nausea and vomiting
  1. Evidence of head injury:
  • Bruising or scalp haematoma
  • Battle’s sign (bruising behind the mastoid process - behind the ear)
  • Periorbital haematoma (Racoon eyes)
How well did you know this?
1
Not at all
2
3
4
5
Perfectly
61
Q

What investigations/tests are used to diagnose extradural haemorrhage?

A

Primary investigations:

Non-contrast CT head: hyperdense well-demarcated biconvex collection below the temporal bone, and the bleed should be limited by the cranial sutures

  • Can also assess for skull fracture:
  • Features of mass effect (not the video game but effect exerted by any mass, presenting as a clinically significant haematoma), such as brain herniation
How well did you know this?
1
Not at all
2
3
4
5
Perfectly
62
Q

What is the management plan for an extradural haemorrhage?

A

An urgent neurosurgical opinion is required.

General principles aim to reduce/control intracranial pressure:

  1. Bed position: have the head of the bed tilted up at ~30 degrees.
  2. Intubation: if the patient has reduced GCS
  3. Oxygen: ensure adequate oxygenation
  4. IV hypertonic saline/mannitol = reduce ICP
  5. Burr hole: temporising measure to reduce ICP before definitive management

Definitive management:

  • Craniotomy and haematoma evacuation:
  • Indicated if the EDH is greater than 30cm3
  • <30cm3 without focal deficit managed non-operatively with serial CT scanning and close neurological observation in a neurosurgical centre
How well did you know this?
1
Not at all
2
3
4
5
Perfectly
63
Q

Define subdural haemorrhage

A

A subdural haemorrhage occurs when blood accumulates between the dura and arachnoid mater (subdural space), forming a subdural haematoma (SDH) collection.

Not a stroke!

How well did you know this?
1
Not at all
2
3
4
5
Perfectly
64
Q

What causes SDH?

A

Subdural haemorrhage commonly occurs after trauma.

The subdural space contains bridging veins which take blood from the brain to the dural venous sinuses. If these veins rupture, they can bleed into the subdural space.

How well did you know this?
1
Not at all
2
3
4
5
Perfectly
65
Q

How are SDH classified?

A

Classified in terms of acuity:

  1. Acute SDH are < 3 days old
    - Caused by traumatic rupture of bridging cortical veins
  2. Chronic SDH are > 21 days old
    - Caused by traumatic rupture of bridging cortical veins

Usually associated with cortical atrophy e.g. elderly and alcoholics > cortical veins more prone to rupture and require lower impact trauma

Cortical atrophy results in larger subdural space, so haematomas can enlarge significantly over time while remaining asymptotic

  1. Subacute SDH are 3-21 days
How well did you know this?
1
Not at all
2
3
4
5
Perfectly
66
Q

What signs and symptoms might a patient with an acute SDH present with?

A

Symptom onset within 3 days of inciting event:

  • May have a lucid interval
  • Reduced consciousness
  • Headaches and vomiting
  • Slurred speech
  • Focal neurological deficit, e.g. weakness or fixed dilated pupil
  • Cushing’s reflex - hypertension, irregular breathing, bradycardia
  • Seizures
How well did you know this?
1
Not at all
2
3
4
5
Perfectly
67
Q

What is Cushing’s reflex?

A

The Cushing reflex is a physiological nervous system response to acute elevations of intracranial pressure (ICP).

This results in the Cushing triad of widened pulse pressure (increasing systolic, decreasing diastolic) (and hypertension), bradycardia, and irregular respirations.

How well did you know this?
1
Not at all
2
3
4
5
Perfectly
68
Q

What signs and symptoms might a patient with chronic subdural haemorrhage/haematoma present with?

A
  • Insidious onset usually in the elderly or alcoholics

Symptoms

  • Progressive confusion and cognitive deficit
  • Headaches and vomiting

Signs

  • Focal neurological deficit, e.g. weakness or fixed dilated pupil
How well did you know this?
1
Not at all
2
3
4
5
Perfectly
69
Q

What is the Glasgow coma scale?

A

A classification system to assess a patient’s level of impaired consciousness and coma

Assess eye-opening, verbal and motor response.

Severe Head Injury = GCS score of 8 or less
Moderate Head Injury = GCS score of 9 to 12
Mild Head Injury = GCS score of 13 to 15

How well did you know this?
1
Not at all
2
3
4
5
Perfectly
70
Q

What investigations/tests are used to diagnose SDH?

A

Primary investigations:

  • Non-contrast CT head: SDH appears as a crescentic collection around the surface of the brain which is not limited by the suture lines of the skull unlike in an extradural haemorrhage.

There may be evidence of mass effect such as midline shift or herniation

  • Acute: hyperdense (bright)
  • Subacute: as the clot matures, the density starts to drop until it is isodense to the brain cortex
  • Chronic: becomes hypodense (dark)
How well did you know this?
1
Not at all
2
3
4
5
Perfectly
71
Q

What are the general principles for the management of SDH?

A

General principles - aim to reduce/control intracranial pressure:

  1. Bed position: have the head of the bed tilted up at ~30 degrees
  2. Intubation: if the patient has reduced GCS
  3. Oxygen: ensure adequate oxygenation
    Maintain adequate cerebral perfusion (Perfusion = MAP - ICP)
  4. Inotropes and vasopressors
  5. Fluids
  6. Osmotherapy: hypertonic saline or mannitol helps shift extracellular fluid and CSF into the vascular compartment, reducing ICP
  7. Burr hole (holes in skull): temporising measure to reduce ICP prior to definitive management
How well did you know this?
1
Not at all
2
3
4
5
Perfectly
72
Q

What is the definitive management for SDH?

A

Definitive management:

Conservative: observation for small acute and chronic SDHs not causing neurological deficits

Surgery: involves craniotomy and evacuation. Indications for acute SDH are as follows:

  • Acute SDH with a thickness > 10mm or a midline shift > 5mm
  • GCS < 9 and continuing decrease
  • Fixed and dilated pupils
  • ICP > 20 mmHg

Chronic SDH = surgical evacuation because of mass effect and symptoms

How well did you know this?
1
Not at all
2
3
4
5
Perfectly
73
Q

Define meningitis

A

Meningitis describes inflammation of the leptomeninges (arachnoid and pia mater) and usually occurs due to a bacterial, viral, or fungal infection.

Bacterial meningitis = acute emergency!

How well did you know this?
1
Not at all
2
3
4
5
Perfectly
74
Q

What is the aetiology of meningitis?

A

The cause can be bacterial, viral or fungal. It can also be caused by trauma, cancer or drugs.

Bacterial:
The most common causes are N. meningitidis and S. pneumoniae in the UK

Viral:
Enteroviruses such as coxsackievirus are the most common cause.

Fungal meningitis:
- Cryptococcus neoformans
- Candida
-Mainly affects immunocompromised patients, very rare in immunocompetent people

How well did you know this?
1
Not at all
2
3
4
5
Perfectly
75
Q

Meningitis: what are the most common causative microorganisms by age group?

A

1) 0 to 3 months:

Group B streptococcus*
E. coli
Listeria monocytogenes

2) 3 months to 6 years:

Neisseria meningitidis
Streptococcus pneumoniae
Haemophilus influenzae

3) 6 months to 60 years:

Neisseria meningitidis
Streptococcus pneumoniae

4) > 60 years:

Streptococcus pneumoniae
Neisseria meningitidis
Listeria monocytogenes

How well did you know this?
1
Not at all
2
3
4
5
Perfectly
76
Q

What are the risk factors for developing meningitis?

A
  • Immunocompromised: children, elderly, HIV patients, chemo patients.
  • Non-immunised: at risk of H. influenza, pneumococcal and meningococcal meningitis
  • Crowded environment: e.g. students living in halls of residence
How well did you know this?
1
Not at all
2
3
4
5
Perfectly
77
Q

What signs and symptoms might a patient with meningitis present with?

A

Symptoms:

  1. Meningism
  • Headache
  • Photophobia
  • Neck stiffness
  1. Fever
  2. Nausea and vomiting
  3. Seizures

Signs:

  1. Kernig’s sign: when the hip is flexed and the knee is at 90°, extension of the knee results in pain
  2. Brudzinski sign: severe neck stiffness causes the hips and knees to flex when the neck is flexed
  3. Petechial or purpuric non-blanching rash: associated with meningococcal disease (N. meningitidis)
  4. Pyrexia
  5. Reduced GCS
How well did you know this?
1
Not at all
2
3
4
5
Perfectly
78
Q

What investigations/tests are used to diagnose meningitis?

A

1) 1st line: Blood cultures: positive in the case of bacterial infection

2) Gold standard: Lumbar puncture (CSF sample taken from L3- L4 and sent for bacterial culture and viral PCR) + CSF analysis.

  • CSF gram stain: S. pneumoniae (gram-positive cocci in chains); N. meningitidis (gram-negative diplococci)
  • CSF culture
  • CSF PCR: useful for viruses such as HSV and VZV (varicella aka chickenpox)
  1. FBC: leukocytosis
  2. CRP: raised inflammatory markers
  3. Blood glucose: required in all patients and for comparison with CSF glucose
How well did you know this?
1
Not at all
2
3
4
5
Perfectly
79
Q

When is a lumbar puncture contraindicated?

A

NICE guidelines state not to perform an LP without consultant instruction if any of the following contraindications are present:

  • Raised intracranial pressure suspected (e.g. papilloedema)
  • Reduced or fluctuating consciousness (GCS < 9 or a drop of ≥ 3)
  • Relative bradycardia and hypertension
  • Focal neurological signs
  • Dilated pupil
  • Shock
  • After convulsions (until stabilised)
  • Coagulation abnormalities, platelet count <100x109/L or on anticoagulants
  • Local infection at the lumbar puncture site
How well did you know this?
1
Not at all
2
3
4
5
Perfectly
80
Q

What are some possible differential diagnoses for meningitis?

A
  • SAH (subarachnoid haemorrhage)
  • Migraine
  • Encephalitis
  • Flu
How well did you know this?
1
Not at all
2
3
4
5
Perfectly
81
Q

What is the management plan for meningitis?

A

1) Empirical therapy = IV cefotaxime (+ amoxicillin if < 3 months or >50 years)

2) Assess GCS

3) Antibiotics for most common bacterial causes:

  • N. meningitidis - IV benzylpenicillin (or cefotaxime)
  • S. pneumonia - IV cefotaxime
  • Haemophilus influenzae meningitis - IV cefotaxime

4) Viral = Acyclovir

How well did you know this?
1
Not at all
2
3
4
5
Perfectly
82
Q

Who needs prophylactic antibiotics for meningococcal meningitis and why?

A

People with close, prolonged contact with patients with meningococcal infections (such as meningitis or septicaemia).

The risk of contracting illness highest within 7 days before the onset of illness and within 7 days after diagnosis.

1st line treatment: single dose 500mg oral ciprofloxacin

How well did you know this?
1
Not at all
2
3
4
5
Perfectly
83
Q

CSF analysis: what results would suggest the presence of bacterial meningitis?

A

Appearance = Cloudy

Protein = High (>1g/L)

Glucose = Low (<50% serum glucose)

White Cell Count = High - 10-5000/mm3 (neutrophils)

Culture = Bacteria

Remember! Interpreting lumbar puncture results is a common exam question.

Bacteria in CSF = release of protein + use up glucose. Viruses don’t use glucose but release small amounts of protein. The immune system releases neutrophils in response to bacteria and lymphocytes in response to viruses.

How well did you know this?
1
Not at all
2
3
4
5
Perfectly
84
Q

CSF analysis: what results would suggest the presence of viral meningitis?

A

Appearance = Clear

Protein = Mildly raised/normal (<1g/L)

Glucose = High (>60% serum glucose)

White Cell Count = High 1000/mm3 (lymphocytes)

Culture = Negative

Remember! LP interpretation = common exam q!

Viruses don’t use glucose and may release only a small amount of protein. Immune system will release lymphocytes in response to viruses

How well did you know this?
1
Not at all
2
3
4
5
Perfectly
85
Q

Define multiple sclerosis (MS)

A

Multiple sclerosis (MS) is an autoimmune, cell-mediated demyelinating disease of the central nervous system.

How well did you know this?
1
Not at all
2
3
4
5
Perfectly
86
Q

What are the risk factors for developing MS?

A
  • Age: most commonly diagnosed in 20-40-year-olds
  • Female gender: x3 more women affected
  • Northern latitudes
  • Vitamin D deficiency
  • Family history: HLA-DR2 is implicated; 30% monozygotic twin concordance
  • Family history of other autoimmune disorders
  • EBV infection
How well did you know this?
1
Not at all
2
3
4
5
Perfectly
87
Q

What is the aetiology of MS?

A

Unclear aetiology but thought to be an interplay between environmental triggers and genetic susceptibility, comprising an inflammatory and degenerative component.

Various factors have been implicated, including infection with EBV, vitamin D deficiency, and HLA association.

How well did you know this?
1
Not at all
2
3
4
5
Perfectly
88
Q

What are the different patterns of MS?

A
  1. Relapsing-remitting:
  • The most common pattern (85% of cases)
    Episodic flare-ups (may last days, weeks or months), separated by periods of remission.
  • 60% of patients develop secondary progressive MS within 15 years
  1. Secondary progressive:
  • Starts with relapsing-remitting course, but symptoms progressively worse with no periods of remission - coordination difficulties, bowel/bladder problems
  1. Primary progressive:
  • 10% patients and common in older patients
  • Symptoms get progressively worse from disease onset with no periods of remission.
How well did you know this?
1
Not at all
2
3
4
5
Perfectly
89
Q

What signs and symptoms might a patient with MS present with?

A

Symptoms:

  • Blurred vision and red desaturation: optic neuritis (swelling that damages the optic nerve) is the most common presenting feature
  • Numbness, tingling and other strange sensations
  • Weakness
  • Bowel and bladder dysfunction
  • Uhtoff’s phenomenon: worsening of symptoms following a rise in temperature, such as a hot bath
How well did you know this?
1
Not at all
2
3
4
5
Perfectly
90
Q

What signs and symptoms might a patient with MS present with (continued)?

A

Signs:

  1. Visual: optic neuritis
  2. Sensory loss: due to demyelination of spinothalamic or dorsal columns
  3. Upper motor neuron signs with spastic (increased muscle tone) paraparesis are common
  4. Cerebellar signs: such as ataxia and tremor (usually in a relapse)
  5. Lhermitte’s phenomenon: electric shock sensation on neck flexion
How well did you know this?
1
Not at all
2
3
4
5
Perfectly
91
Q

What investigations and tests are used to diagnose MS?

A
  1. MRI brain and spine:

Demyelinating plaques: Dawson’s fingers, perpendicular to the lateral ventricles > suggestive of MS

  1. Diagnostic MRI brain and spine + McDonald’s criteria (≥ 2 relapses)

Old lesions will not enhance with contrast, whereas newer lesions will. This provides evidence of the dissemination of lesions in time (separate events) and space (different parts of CNS affected), which is required for a diagnosis of MS

  • Lumbar puncture: oligoclonal bands found in the CSF and not in the serum (unmatched), indicating increased IgG.
How well did you know this?
1
Not at all
2
3
4
5
Perfectly
92
Q

What is the management plan for MS?

A
  1. Relapse:
  • Steroids: reduce relapse duration,1st line: oral or IV methylprednisolone for 5 days

2) Maintenance therapy: reduce the relapse rate, for patients 2 relapses in 2 years or progression on MRI

1st line disease-modifying drugs:

  • Beta-interferon: reduces relapse rate, but not progression
  • Glatiramer acetate: immunomodulator drug which might kill immune cells that attack myelin
How well did you know this?
1
Not at all
2
3
4
5
Perfectly
93
Q

Define Guillain-Barre syndrome

A

Guillain-Barré syndrome (GBS) is an autoimmune, rapidly progressive demyelinating condition of the peripheral nervous system, often triggered by infection.

How well did you know this?
1
Not at all
2
3
4
5
Perfectly
94
Q

What is pathophysiology of GBS?

A

GBS is believed to be caused by ‘molecular mimicry.’

A pathogenic antigen (e.g. Campylobacter jejuni) resembles myelin gangliosides in the PNS.

The immune system targets the antigen and attacks the myelin sheath of sensory and motor nerves.

This autoimmune process involves the production of anti-ganglioside antibodies (anti-GMI +ve 25% of patients)

How well did you know this?
1
Not at all
2
3
4
5
Perfectly
95
Q

What is the main type of Guillian-Barre Syndrome?

A

1) Acute inflammatory demyelinating polyneuropathy (ADIP): ~90% of cases

How well did you know this?
1
Not at all
2
3
4
5
Perfectly
96
Q

DDx for Guillain-Barre Syndrome

A
  • Myasthenia Gravis
  • Polymyositis
How well did you know this?
1
Not at all
2
3
4
5
Perfectly
97
Q

What signs and symptoms might a patient with GBS present with?

A

Typical presentation - symmetrical muscle weakness affecting lower extremities before upper (ascending weakness). History of infection within the preceding 6 weeks

Subacute, with peak symptoms within 2-3 weeks of disease onset

How well did you know this?
1
Not at all
2
3
4
5
Perfectly
98
Q

What signs and symptoms might a patient with GBS present with (continued)?

A

Signs:

  • Reduced sensation in affected limbs - mild on examination
  • Symmetrical weakness in lower extremities first, progressing to the upper limbs: proximal muscles affected before distal muscle
  • Ataxia with hyporeflexia (or areflexia - absent deep tendon reflexes) in affected limbs
How well did you know this?
1
Not at all
2
3
4
5
Perfectly
99
Q

What investigations and tests are used to diagnose GBS?

A

Mainly a clinical diagnosis - progressive weakness and areflexia/hyporeflexia in weaker limbs.

  • Diagnostic lumbar puncture: raised CSF protein with normal WBC count is typical
  • Nerve conduction studies - suggestive of demylintion e.g. reduced conduction velocity
  • LFTs: raised AST (aspartate aminotransferase) and ALT (alanine aminotransferase)
  • Spirometry: shows reduced vital capacity + used to monitor respiratory function
How well did you know this?
1
Not at all
2
3
4
5
Perfectly
100
Q

What is the management plan for GBS?

A

1st line:

IV immunoglobulins (IVIg): 5 day treatment course started within 2 weeks of symptom onset

OR

Plasma exchange - over 2 weeks within 4 weeks of symptom onset

How well did you know this?
1
Not at all
2
3
4
5
Perfectly
101
Q

Define Parkinson’s disease

A

Parkinson’s Disease (PD) is a neurodegenerative disorder characterised by loss of dopaminergic neurones within the substantia nigra pars compacta (SNPC) of the basal ganglia (nigrostriatal pathway).

How well did you know this?
1
Not at all
2
3
4
5
Perfectly
102
Q

What is the aetiology of PD?

A

Idiopathic, aetiology not clear.

Key genes involved are abnormalities in those that code for α-synuclein and the ubiquitin-protease system.

This results in a loss of transmission between the basal ganglia, thalamus, and motor cortex, resulting in impaired control of voluntary movements.

How well did you know this?
1
Not at all
2
3
4
5
Perfectly
103
Q

What is the pathophysiology of PD?

A

Histological hallmark - eosinophilic inclusion bodies (Lewy bodies) consisting of misfolded α-synuclein in the dopaminergic neurones

The effect of these misfolded proteins is still unclear but thought that misfolded α-synuclein may spread to neighbouring brain regions in a prion-like fashion

How well did you know this?
1
Not at all
2
3
4
5
Perfectly
104
Q

What are the risk factors for developing PD?

A

Parkinson’s Disease is the second most common neurodegenerative disease (after Alzheimer’s Disease):

  • Age: prevalence of 1% in 60-70 and around 1-3% in ≥80
  • Gender: males are x1.5 more likely to develop PD
  • Family History
How well did you know this?
1
Not at all
2
3
4
5
Perfectly
105
Q

What signs and symptoms might a patient with PD present with?

A

Parkinsonism:

  • Bradykinesia
  • Tremor (resting 3.5Hz, distal, reduce on action)
  • Rigidity
  • Postural instability (causing shuffling gait with short, rapid steps (festination))

Non-motor symptoms:

  • Anosmia
  • Sleep disturbance
  • Psychiatric symptoms including: depression, anxiety, dementia
  • Constipation
How well did you know this?
1
Not at all
2
3
4
5
Perfectly
106
Q

What are some DDx for parkinsonism?

A
  • Lewy-body dementia (if dementia symptoms onset before parkinsonism)
  • Wilson’s disease
  • Meds (e.g. antipsychotics)
  • CNS infections - following encephalitis
How well did you know this?
1
Not at all
2
3
4
5
Perfectly
107
Q

What investigations/tests are used to diagnose PD?

A
  1. 1st line - Mainly clinical diagnosis for typically presenting patients
  2. Investigations to consider if atypical features or unclear clinical diagnosis
  • MRI brain: may help exclude other causes of neurological disease but should not be used to diagnose PD
  • SPECT (DaT scan): single-photon emission computed tomography (SPECT) will show reduced dopamine uptake in the basal ganglia
How well did you know this?
1
Not at all
2
3
4
5
Perfectly
108
Q

What is the first-line management plan for PD?

A

NICE recommends that patients with suspected PD should be referred to a specialist in movement disorders (either a neurologist or elderly care physician) for a diagnosis to be made.

  • Confirmed diagnosis should be reviewed every 6-12 months

1st line:

  1. Motor symptoms affecting quality of life:
  • Levodopa (dopamine replacement) + decarboxylase inhibitor e.g.
    Co-beneldopa (combo reduces/stops N+V caused by Levodopa)
  1. Motor symptoms not affecting quality of life
  • A choice of one of the following:
  • Dopamine agonist: e.g. ropinirole
  • Monoamine oxidase B inhibitor: e.g. selegiline
  • Levodopa + decarboxylase inhibitor
    Co-beneldopa
How well did you know this?
1
Not at all
2
3
4
5
Perfectly
109
Q

Define Huntington’s disease

A

Progressive neurodegenerative disorder with 100% penetrance (proportion of carriers of a gene exhibiting an associated phenotype).

It is an autosomal dominant, trinucleotide repeat disorder caused by a loss of the main inhibitory neurotransmitter GABA.

How well did you know this?
1
Not at all
2
3
4
5
Perfectly
110
Q

What is the aetiology of Huntington’s disease?

A

Huntington’s disease is a trinucleotide repeat disorder caused by mutation in the huntingtin (HTT) gene (on chromosome 4), causing expanded CAG repeats at the N-terminus of the gene that codes for the huntingtin protein.

CAG = glutamine

≤35 CAG repeats = normal
36 - 39 CAG repeats = reduced penetrance

≥40 CAG repeats = certain disease development

Average age of onset around 40, but depends on number of CAG repeats

How well did you know this?
1
Not at all
2
3
4
5
Perfectly
111
Q

What is the pathophysiology of Huntington’s disease?

A

The polyglutamine tail on the HTT protein is cleaved and forms toxic fragments > causing neuronal cell death in the putamen and caudate nuclei (dorsal striatum) of basal ganglia, which have a role in movement inhibition and rely on GABA as its main neurotransmitter.

Results in loss of GABA > less regulation of dopamine > excess dopamine > symptoms of Huntington’s (see signs and symptoms card)

How well did you know this?
1
Not at all
2
3
4
5
Perfectly
112
Q

What signs and symptoms might a patient with Huntington’s disease present with?

A
  • Chorea - jerky, dance-like movements
  • Dystonia - muscle spasms and contractions
  • Incoordination
  • Cognitive and behavioural difficulties
  • Irritability, agitation and anxiety
How well did you know this?
1
Not at all
2
3
4
5
Perfectly
113
Q

Explain anticipation in terms of the inheritance of Huntington’s disease.

A

Anticipation is a phenomenon in which the signs and symptoms of some genetic conditions tend to become more severe and/or appear at an earlier age as it is passed from one generation to the next.

In terms of HD, the repeat expansion of CAG also affects DNA replication, with CAG repeats added during DNA replication. The greater the number of repeats, the more severe the disease. This affects sperm more so than eggs,

This means the child of a parent with HD can inherit even more CAG repeats than the parent did > earlier age of symptoms onset, this is anticipation > HD manifest earlier with each generation

How well did you know this?
1
Not at all
2
3
4
5
Perfectly
114
Q

What investigations/tests are used to diagnose HD?

A
  1. Clinical diagnosis
  2. Diagnosis confirmation = CAG repeat test:
  • A positive result is ≥40 CAG repeats on 1 of the 2 alleles; an intermediate result is 36 to 39 repeats
  1. MRI/CT = loss of striatal volume/striatal atrophy
How well did you know this?
1
Not at all
2
3
4
5
Perfectly
115
Q

What is the management plan for HD?

A
  1. 1st line is counselling for patients and family
  2. Benzodiazepines, dopamine-depleting agents (e.g. tetrabenazine), antipsychotics (e.g. haloperidol) for chorea
  3. SSRI for depression
How well did you know this?
1
Not at all
2
3
4
5
Perfectly
116
Q

What is a DDx for chorea?

A

Sydenham’s chorea ( Rheumatic fever)

How well did you know this?
1
Not at all
2
3
4
5
Perfectly
117
Q

What is the prognosis of HD?

A
  • Poor prognosis - death within 10 - 20 years of diagnosis

The most common cause of death = aspiration pneumonia

Second most common cause = suicide

How well did you know this?
1
Not at all
2
3
4
5
Perfectly
118
Q

Define encephalitis

A

Encephalitis describes inflammation of the brain parenchyma.

How well did you know this?
1
Not at all
2
3
4
5
Perfectly
119
Q

What is the aetiology of encephalitis?

A

The most common cause of encephalitis is herpes simplex virus-1 (HSV-1; 95% of cases), which causes temporal and inferior frontal lobe encephalitis.

Other causes:
- EBV
- CMV
- HIV
- TB
- Toxoplasmosis (close contact with cats)

How well did you know this?
1
Not at all
2
3
4
5
Perfectly
120
Q

What are the risk factors for developing encephalitis?

A
  • Immunocompromise (elderly, babies, HIV)
  • Blood/fluid exposure: HIV and West Nile virus
  • Mosquito bite: West Nile virus
  • Transfusion and transplantation: CMV, EBV, HIV
  • Close contact with cats: toxoplasmosis
How well did you know this?
1
Not at all
2
3
4
5
Perfectly
121
Q

What signs and symptoms might a patient with encephalitis present with?

A

Symptoms:

  1. Fever
  2. Headache
  3. Behavioural changes:
  • Memory disturbance
  • Psychotic behaviour
  • Withdrawal or change in personality
  1. Seizures

Signs:

  1. Pyrexia
  2. Reduced GCS
  3. Focal neurological deficit, such as:
  • Aphasia
  • Hemiparesis (weakness of one side of the body)
  • Cerebellar signs
How well did you know this?
1
Not at all
2
3
4
5
Perfectly
122
Q

What investigations/tests are used to diagnose encephalitis?

A

Blood tests: FBC (high WBC count), CRP, U&Es (hyponatremia) and blood culture (can detect bacterial infection)

Throat swab: culture for viral organisms

CT or MRI (preferred) head: inflammation in the temporal and inferior frontal lobes in HSV encephalitis

CSF analysis: lymphocytosis (high lymphocyte) with raised protein in the case of viral aetiology

PCR: assays for common viral infections
including HSV

How well did you know this?
1
Not at all
2
3
4
5
Perfectly
123
Q

What is the management plan for encephalitis?

A

Antiviral medication:

Aciclovir - empirical treatment in all patients with suspected encephalitis caused by HSV-1

How well did you know this?
1
Not at all
2
3
4
5
Perfectly
124
Q

Define dementia

A

Dementia, is a progressive decline in cognitive function affecting multiple domains, including language, executive function, memory and social cognition

How well did you know this?
1
Not at all
2
3
4
5
Perfectly
125
Q

What are the main types of dementia?

A
  1. Alzheimer’s Disease (50-75%)
  2. Vascular Dementia (up to 20%)
  3. Dementia with Lewy-Bodies (10-15%)
  4. Frontotemporal Dementia (2%)
How well did you know this?
1
Not at all
2
3
4
5
Perfectly
126
Q

What is the pathophysiology of Alzheimer’s Disease?

A

Deposition of extracellular 𝛃-amyloid (senile plaques) and intracellular tau protein (neurofibrillary tangles) lead to neurotoxicity and reduced cholinergic transmission.

127
Q

What is the pathophysiology of vascular dementia?

A

Reduced blood flow to the brain, secondary to small or large vessel disease.

128
Q

What is the pathophysiology of Lewy body dementia?

A

Cortical and subcortical deposition of Lewy-Bodies (intracellular aggregates of ɑ-synuclein)

129
Q

What is the pathophysiology of frontotemporal dementia?

A

The progressive degeneration of the frontal and/or temporal lobes. associated with Pick bodies, with several subtypes such as Pick’s disease (build-up of tau proteins in neurons, accumulating into silver-staining, spherical aggregations i.e. Pick bodies)

130
Q

What are the risk factors for developing dementia?

A
  • Age: risk increases with age
  • High BMI - increased inflammatory processes
  • Smoking: results in vascular changes as well as inflammation and oxidative stress
  • Type 2 diabetes: vascular changes
  • Hypertension: vascular changes
  • Depression
  • Low education
  • Hearing loss: from reduced cognitive stimulation
  • Social isolation and loneliness: reduced cognitive stimulation, and associated with poorer health outcomes
131
Q

What are the clinical features of Alzheimer’s disease?

A
  • Characteristic order of language impairment: naming → comprehension → fluency
  • Memory impairment
132
Q

What are the clinical features of vascular dementia?

A
  • Evidence of previous stroke
  • Depression and delusions
  • Emotional lability
  • Memory impairment
  • Gait disturbance and incontinence are sometimes seen.
133
Q

What are the clinical features of Dementia with Lewy-Bodies?

A
  • Cognitive symptoms precede motor symptoms unlike in Parkinson’s related dementia
  • Visual hallucinations and delusions
  • REM sleep disorder
  • Memory and attention impairment
  • Parkinsonism
134
Q

What are the clinical features of frontotemporal dementia?

A
  • Behavioural and personality changes, e.g. impulsivity
  • Memory impairment
  • Language impairment e.g. reduced fluency or comprehension - develop a lot quicker than other forms
135
Q

What investigations/tests are used to diagnose dementia?

A

No specific tests for dementia, so investigations can focus on two factors to guide diagnosis:

1) Observing a change in behaviour over a period of time using:

  • History: incorporate cognitive, behavioural and psychological symptoms and impact on daily life
  • Cognitive assessments using a validated tool

Most common: Folstein Mini-Mental State Examination (MMSE) - 11 questions testing orientation, attention, memory, language and visual-spatial skills

A score of <24 out of 30 = abnormal

21 - 25 = reevaluation in 3 - 6 months

MMSE should be combined with history and decline from the baseline level of functioning, as it might not detect mild cognitive decline.

2) Excluding reversible causes of cognitive decline:

  • Medication review: anticholinergics, opioids, benzodiazepines
  • Bloods: FBC (anaemia), U&Es (abnormal Na, Ca, glucose levels), TFTs (hypo/hyperthyroid), vitamin B12 (pernicious anaemia)
  • CT or MRI head to exclude a structural lesion
136
Q

What is the conservative management plan for dementia?

A
  • Memory service: MDT involved in diagnosing and supporting patients
  • Education: education about dementia and adapting to changes
  • Advanced care plan: power of attorney, advanced statement on patient’s wishes, preferences to place of care and death
  • Cognitive stimulation therapy: activities and discussions aimed at improving cognitive and social functioning
  • Group reminiscence therapy: uses objects from daily life to stimulate memory and enable people to value their experiences.
  • Medication review
137
Q

What is the medical management plan for Alzheimer’s disease?

A

Mild/moderate disease:

1st line: AChE (Acetylcholinesterase) inhibitors: e.g. donepezil

Moderate/severe disease:
Memantine: NMDA receptor (glutamate as excitory neurotransmitter) antagonist in combination with an AChE inhibitor (but can be monotherapy if the former isn’t tolerated)

138
Q

What is the medical management plan for vascular dementia?

A
  1. Optimise cardiovascular risk factors: to reduce the likelihood of further ischaemic events
  2. AChE inhibitor or memantine (glutamate receptor antagonist): if there is suspected comorbid Alzheimer’s disease, Dementia with Lewy-Bodies or Parkinson’s disease
139
Q

What is the medical management plan for Lewy body dementia?

A

1st line: AChE (Acetylcholinesterase) inhibitor: donepezil or rivastigmine

Dopamine replacement (levodopa and carbidopa): if there is evidence of parkinsonism

Atypical antipsychotics if severe psychiatric symptoms, but might worsen motor symptoms (avoid as far as possible)

140
Q

What is the medical management plan for frontotemporal dementia?

A
  • AChE inhibitors and memantine are not recommended
  • Conservative measures is recommended (see conservative management plan for dementia)
141
Q

What are some primary causes of a headache?

A
  • Migraines
  • Tension headaches
  • Cluster headaches
142
Q

What are some secondary causes of a headache?

A
  • Giant cell/temporal arteritis
  • Thunderclap headache (subarachnoid haemorrhage)
  • Trauma
  • Medication overuse
  • Trigeminal neuralgia
  • Systemic infection
  • Meningitis/encephalitis
143
Q

What are some red flags for headaches?

A
  • Worst headache ever
  • New onset headache > 50
  • Severe, quick onset
  • Abnormal pattern of migraine
  • Progressive or persistent headaches, or headaches that have changed dramatically
144
Q

Define migraine

A

Migraine is a primary headache of complex aetiology which is usually episodic. ‘Classic’ migraines are preceded by an aura, however, these only occur in one-third of patients.

145
Q

Types of migraines

A
  • Migraine with typical aura
  • Migraine with atypical aura
  • Hemiplegic migraine
  • Silent migraine (migraine with aura but without a headache)
146
Q

What is the aetiology of migraines?

A

Triggers: CHOCOLATE

  • CHocolate
  • Oral Contraceptive
  • Alcohol + Anxiety
  • Travel
  • Exercise
  • Other important triggersinclude tiredness, lack of food, dehydration, menstruation, red wine and bright lights
147
Q

What is the pathophysiology of migraines?

A
  • Neuronal hyperexcitability > trigeminal nerves initiates an inflammatory response > dilation of meningeal blood vessels, and sensitisation of surrounding nerve fibres = pain
  • The aura is thought to occur due to cortical spreading depression, a wave of depolarisation across the cerebral cortex.
148
Q

What signs and symptoms might a patient with migraine present with?

A
  1. Prodrome (days prior attack)
    Yawning, cravings, mood and sleep changes
  2. Aura (typical or atypical) - before an attack
  3. Clinical presentation
  • Unilateral pain but can be bilateral
  • Throbbing/pounding pain
  • Moderate to severe intensity
  • Nausea/ vomiting
  • Photophobia + phonophobia
149
Q

Features of a typical aura

A

Develops over 5 minutes, lasts 5-60 minutes and is fully reversible

  • Visual disturbances ( lines, dots, zigzags)
  • Somatosensory ( Paraesthesia, pins and needles)
  • Speech disturbances
150
Q

What symptoms/signs might a migraine with atypical aura present with?

A

May last more than 60 minutes

  • Motor weakness (e.g. hemiplegic migraine)
  • Diplopia
  • Visual symptoms affecting one eye
  • Poor balance (e.g. vestibular migraine)
  • Decreased level of consciousness

Need urgent investigation to to rule out other sinister causes such as a stroke. Particularly for hemiplegic migraine, where patients can present with unilateral weakness and headache and can therefore easily be mistaken for a stroke.

151
Q

What investigations/tests are used to diagnose migraines?

A

Diagnosis is based on history and physical examination. No laboratory or imaging tests are essential for diagnosis.

The diagnostic criteria is the International Classification of Headache Disorders (ICHD)-3 criteria for migraine.

152
Q

What is the management plan for migraines?

A
  1. Conservative management: avoid triggers
  2. Acute:
  • Analgesia: ibuprofen or aspirin (900mg) or paracetamol
  • Mild: analgesia
  • Severe = oral triptans and analgesia e.g sumatriptan is 1st line
  • Antiemetic: metoclopramide

3) Chronic:

  • Headache diary
  • Avoid triggers
  • Prophylaxis - Propranolol is considered first-line

4) Prophylaxis (non-pharmacological):

  • Mindfulness
  • Acupuncture
  • Riboflavin (B2): may be effective in some people but avoid in pregnancy
153
Q

Define tension headache

A

Most chronic and recurrent daily headache, Most common primary headache.

Classically they produce a mild ache across the forehead and in a band-like pattern around the head.

Tension headaches comes on and resolve gradually and don’t produce visual changes.

154
Q

What can trigger a tension headache?

A
  • Missed meals
  • Conflict
  • Stress
  • Clenched jaw
  • Overexertion
  • Fatigue
  • Depression
  • Stress
  • Bad posture
  • Hunger
  • Noise
155
Q

What signs and symptoms might patients with tension headaches present with?

A
  • Bilateral, pressing headache ( tight band)
  • Not aggravated by movement
  • 30 mins to 7 days
  • No nausea + vomiting
  • No more than one out of photophobia / phonophobia
  • Scalp tenderness
156
Q

What are some DDx for tension headaches?

A
  • Migraine
  • Cluster headache
  • GCA (giant cell arthritis)
  • Drug-induced headache ( worsens on analgesia
157
Q

What is the management plan for tension headaches?

A
  • Reassurance
  • Stress relief
  • Medication = Paracetamol , NSAIDs (Ibuprofen, diclofenac)
  • TCA’s
  • Analgesia no more than 6 days per month
158
Q

Define cluster headaches

A
  • Cluster headache is an attack of severe pain localised to the unilateral orbital, supra-orbital, and/or temporal areas that lasts from 15 minutes to 3 hours.
  • It is the most painful and disabling primary headache. It is rare and more common in males
  • Onset between 20 and 40
159
Q

What are the risk factors for developing cluster headaches?

A
  • Male
  • Smoker
  • Alcohol
  • Genetics
160
Q

What are the signs and symptoms of a cluster headache?

A
  • Rapid onset of excruciating pain generally localised around the eye
  • Unilateral rising to a crescendo over a few minutes and lasting between 15 to 160 mins, once or twice a day generally around the same time
  • Watery, bloodshot eye
  • Facial flushing
  • Rhinorrhoea
  • Miosis ( pupillary constriction ) / ptosis (droopy eyelid)
161
Q

What investigations/tests are used to diagnose cluster headaches?

A
  • Brain MRI - normal in primary cluster headaches, abnormal results might suggest secondary cause e.g. tumours.
  • ESR to exclude giant cell arteritis in patients over 50 years of age. Normal in primary cluster headaches
  • Pituitary function tests - normal in primary cluster headache; abnormalities may suggest secondary causes resulting from a pituitary adenoma
  • Diagnosis is based on the International Headache Society International Classification of Headache Disorders, 3rd edition (ICHD-3) criteria.
162
Q

What is the management plan for cluster headaches?

A

Acute
1) No analgesia

2) 1st line: 12L/min 100% oxygen for 15 mins via a non-rebreather mask (stops expired air going back into the oxygen reservoir)

3) Triptans - 1st line = Sumatriptan

4) Prophylaxis - 1st line = verapamil (CCB)

163
Q

Define trigeminal neuralgia

A

Trigeminal neuralgia is a pain syndrome which describes severe unilateral pain in the distribution of one or more trigeminal branches (ophthalmic, maxillary and mandibular zone)

164
Q

What is the aetiology of trigeminal neuralgia?

A

90% of trigeminal neuralgia cases have compression of the trigeminal nerve typically by the superior cerebellar artery.

Other causes include demyelinating disease (MS patients x20 more likely), posterior fossa masses, and brainstem infarcts.

165
Q

What are the risk factors for developing trigeminal neuralgia?

A
  • Advancing age: >40
  • Female gender
  • Demyelinating disease: 20 times more common in patients with MS
166
Q

What signs and symptoms might a patient with trigeminal neuralgia present with?

A

Symptoms:

As per NICE, the pain is as follows:

  • Trigeminal distribution (ophthalmic, maxillary and mandibular zones)
  • Severe
  • Unilateral
  • Electric shock-like sensation
  • Seconds to minutes
  • Episodic
  • Provoked, e.g. touch or cold
  • Some patients experience autonomic symptoms such as lacrimation, facial swelling, rhinorrhoea or ptosis

Signs:

  • Pain may be provoked by touch on examination
167
Q

What investigations/tests are used to diagnose trigeminal neuralgia?

A

Trigeminal neuralgia is a clinical diagnosis but further investigations may be considered to rule out alternative pathology.

Consider:
MRI brain: if a sinister cause is suspected e.g. space-occupying lesion or demyelination, or if the patient is refractory to medical treatment and surgery is being considered

168
Q

What is the management plan for trigeminal neuralgia?

A

1st line
Medical: carbamazepine (anticovulasant)

The dose is titrated upwards every 2 weeks until the pain is relieved.

Refer to neurology: if there is severe pain or pain that significantly affects daily function, as well as patients refractory to treatment or with atypical symptoms (e.g. age < 50 years)

169
Q

DDx for trigeminal neuralgia

A
  • Dental caries
  • Dental fracture
  • Mandibular osteomyelitis
  • Migraine
170
Q

Define epilepsy

A

Epilepsy is a neurological disorder characterised by recurrent seizures.

A seizure = paroxysmal alteration of neurological function resulting from the excessive, hypersynchronous discharge of neurons (a lot of neurons firing simultaneously) within the brain.

Main excitatory neurotransmitter - glutamate, NMDA receptor

The disrupted neurological function causing a seizure is the result of an imbalance between excitation and inhibition within the neurons of the brain.

171
Q

What are the risk factors for developing epilepsy?

A

Epilepsy can be inherited or acquired.

Inherited:

  • Premature birth
  • Genetic condition: cerebral palsy: up to 30% have epilepsy etc.
  • Family history

Acquired:

  • Traumatic brain injury
  • Neurodegenerative disease (Alzheimer’s x10 more likely)
  • Ischaemic stroke
  • Intracerebral infection: meningitis
  • Illicit drug use: cocaine
172
Q

What are the categories of epilepsy?

A
  • Focal: originates within one side of the brain, starting in the temporal lobes.
  • Generalised: involve both hemispheres of the brain at onset
  • Focal to bilateral: previously termed ‘secondary generalised’, the seizure starts on one side of the brain and spreads to both sides
  • Unknown: difficult to determine in some cases
173
Q

What are the subcategories of focal seizures?

A
  • Motor
  • Non-motor
  • With retained awareness
  • Impaired awareness
174
Q

Focal seizures: what is a focal motor seizure?

A

Caused by distorted neuronal activity in the brain lobes involved in motor control (usually the frontal lobe).

Symptoms include jerking, twitching or stiffening of a body part or automatisms such as licking lips, rubbing hands, walking

175
Q

Focal seizures: what is a focal non-motor seizure?

A

Seizures that cause changes in sensation, emotions, thinking or experiences

176
Q

Focal seizures: what is a focal seizure with retained awareness?

A

Focal seizures with retained awareness are when the patient is aware during the seizure, even if they cannot speak or respond

177
Q

Focal seizures: what is a focal seizure with impaired awareness?

A

The patient is not aware when they are having a focal seizure

178
Q

What are the subcategories of generalised seizures?

A
  • Tonic-clonic
  • Tonic
  • Clonic
  • Myoclonic*
  • Atonic*
  • Generalised non-motor (absence)

*Can be either generalised or focal

179
Q

Generalised seizures: what is a tonic-clonic seizure?

A

Stiffening of limbs (tonic) with rhythmic jerking (clonic) simultaneously (most people recognise this)

180
Q

Generalised seizures: what is a tonic seizure?

A

Sudden stiffness or tension in muscles of arm, legs or trunk.

181
Q

Generalised seizures: what is a clonic seizure?

A

Repeated jerking movements of arms or legs on one or both sides of the body.

182
Q

Generalised seizures: what is a myoclonic seizure?

A

Sudden, short-lasting jerks that can affect some or all of your body, usually lasts less than a second.

Ranges from mild (a twitch) to forceful, which can cause the patient to throw things they are holding.

183
Q

Generalised seizures: what is an atonic seizure?

A

Sudden loss of muscle strength. Can cause a person to drop to the ground.

184
Q

Generalised seizures: what is an generalised non-motor (absence) seizure?

A

A seizure that involves brief changes in awareness - typically involves behavioural arrest or staring lasting 5 to 10 seconds

185
Q

What lobes of the brain can focal and focal to bilateral seizures affect? Give the function of each lobe and a symptom that a seizure can cause

A

1) Temporal - memory, hearing and understanding language - hearing ringing/buzzing. Automatisms like lip-smacking, picking, walking

2) Frontal - motor control - twitching, jerking, stiffening of one part of your body

3) Parietal - spatial perception, touch, language understanding etc. - paraesthesia

4) Occipital - processing of visual info - visual hallucinations, transient blindness,
rapid and forced blinking

186
Q

What symptoms does a patient with a focal seizure affecting the temporal lobe present with?

A

Automatism: lip smacking, chewing, fumbling etc.

Automatic behaviour: running, walking

Auras: déjà vu, feeling of fear, unpleasant smells

Auditory: buzzing, ringing, vertigo

187
Q

What symptoms does a patient with a focal to bilateral seizure affecting the frontal lobe present with?

A

Predominantly motor symptoms: thrashing, twitching, jerking

188
Q

What symptoms does a patient with a focal to bilateral seizure affecting the parietal lobe present with?

A
  • Parasthesias
  • Visual hallucinations
  • Visual illusions
189
Q

What symptoms does a patient with a focal to bilateral seizure affecting the occipital lobe present with?

A
  • Visual hallucinations
  • Transient blindness
  • Rapid and forced blinking
  • Movement of head or eyes to the opposite side
190
Q

Describe the components of an epileptic seizure

A

1) Prodrome

  • Precedes seizure by hours to days
    Weird feelings ( mood, behavioural changes)

2) Aura

  • Patient is aware + precedes other manifestations
  • Strange feelings in the gut, deja vu, strange smells, flashing lights
  • Can occur before a partial seizure

3) Ictal - seizure

4) Postictal

  • After seizure
  • Headache, confusion, myalgia, sore tongue ( often bitten)
  • Temporary weakness after focal seizure in motor cortex = Postictal Todd’s palsy
  • Dysphasia after temporal lobe seizure
191
Q

What criteria are used to diagnose epileptic seizures

A

Criteria for diagnosis: any one of the following:

  • 2 unprovoked seizures occurring > 24 hours apart
  • One unprovoked seizure + probability of future seizures e.g. abnormal EEG reading (> 60%)

If the patient fulfills one of the above, then the epileptic syndrome diagnosis.

192
Q

What investigations (apart from the criteria mentioned above) are used to diagnose epileptic seizures?

A

Electroencephalogram (EEG) - generalised epileptiform activity, or focal localising abnormality

Investigations to exclude other causes.

  • Capillary blood glucose: to exclude hypo/hyperglycaemia as they can cause tonic-clonic seizures
  • FBC: leukocytosis could indicate CNS infection, which provoke generalised tonic-clonic seizures
  • U&Es - severe electrolyte imbalances (hyponatraemia, hypernatraemia, or uraemia) can cause generalised tonic-clonic seizures
  • Toxicology screen: if drug-induced seizure suspected
  • Imaging:CT head: to exclude intracranial causes such as haemorrhage

All adults presenting with a seizure should be referred urgently to a specialist in the management of epilepsy (usually via a ‘first fit’ clinic) to ensure accurate and early diagnosis and initiation of appropriate therapy.

DIagnosis by neurologist using combination of history and investigations.

193
Q

What are some causes of seizures apart from epilepsy?

A

Fever, hypoglycaemia or alcohol withdrawal.

These “provoked” seizures above are caused by a specific substance/disease are reversible and do not fall under the diagnosis of epilepsy which is unprovoked.

194
Q

What features can help differentiate non-epileptic seizures from epileptic seizures?

A

Non-epileptic seizures:

  • Caused by fever, alcohol withdrawal, metabolic disturbances ( hyponatremia, hypoxia, hypoglycemia)
  • Lasts longer ( > 2 -3 mins), epileptic seizure ≤ 15s
  • Does not occur in sleep
  • No incontinence + tongue biting
  • No muscle pain
195
Q

What is the management plan for epileptic seizures - conservative?

A

Conservative, medical or surgical

Conservative:

  • Education: triggers to avoid, safety measures

Stop driving: all patients must inform the DVLA:

  • First unprovoked seizure without structural abnormality or epileptiform activity on EEG: 6-month suspension
  • First unprovoked seizure with a structural abnormality or epileptiform activity on EEG: 12-month suspension
196
Q

What are the medical treatments/management for epileptic seizures?

A

Anti-epileptic drugs (AEDs):

  • Generalised = sodium valproate (teratogenic - DO NOT PRESCRIBE to women who are able to have children)
  • Focal = lamotrigine or levetiracetam
  • Female of childbearing potential = Lamotrigine
  • Myoclonic = Levetiracetam / Topiramate
  • Absence = Ethosuximide
197
Q

What are the surgical treatments/management for epileptic seizures

A

If medical management fails

  • Focal resection (removing brain tissue where the seizures originate)

Lobe resection (removal of part of the temporal lobe)

Lesionectomy: if a specific structural lesion is determined

198
Q

What is status epilepticus?

A

A seizure is hypersynchronous, abnormal neuronal activity occurring in the brain.

Status epilepticus (SE) is a single, continuous seizure lasting more than five minutes or two or more seizures within five minutes without regaining consciousness in between.

It is a complication of epilepsy and medical emergency.

199
Q

What is the treatment/management plan for status epilepticus?

A
  • ABCDE approach (Airway, Breathing, Circulation, Disability, Exposure) + high-flow oxygen
  • 1st line: 4mg IV Lorazepam, if ineffective then Lorazepam again then phenytoin
200
Q

Define spinal cord compression (SCC)

A

Spinal cord compression (SCC) is as a result of trauma to the spinal cord and can cause injury to the spinal cord due to external pressure.

This damages the white and grey matter and causes loss of all or some of the sensory modalities and motor functions.

Spinal cord compression/injury affects the spinal cord (above L1 - L2). For SCC affecting regions below L1 - 2, it is known as cauda enquina syndrome.

201
Q

What are the most common causes of SCC?

A
  • Trauma: car accidents, falls
  • Vertebral compression fractures: osteoporosis, osteomalacia
  • Intervertebral disc disease: disc herniation
  • Tumours: metastatic disease, CNS tumours
  • Infection: osteomyelitis
202
Q

What signs and symptoms might a patient with SCC present with?

A

Symptoms:

1) Acute onset: more likely trauma or disc herniation

2) Insidious onset: more likely malignancy, osteoporosis

  • Back pain
  • Paresthesia
  • Weakness
  • Bladder or bowel dysfunction

Signs:

1) Upper motor neuron weakness below lesion (often presents after spinal shock):

  • Loss of muscle power
  • Increased tone
  • Hypereflexia

2) Sensory deficit: fine-touch, vibration, temperature, (affecting dorsal column medial lemniscus (fine-touch) and spinothalamic tracts (pain + temp, crude touch + pressure)

3) Spinal shock:

  • Hypo- or areflexia below the level of injury
  • Motor paralysis below the level of injury

4) Neurogenic shock:

  • Typically, cervical/high thoracic injury
  • Bradycardia
  • Peripheral vasodilation
  • Decreased cardiac output
  • Priapism (prolonged erection)
203
Q

What are the different patterns of SCC?

A

1) Complete spinal cord injury

2) Central cord syndrome (usually affects the cervical spine)

3) Anterior cord syndrome (disruption of anterior spinal cord/artery)

4) Posterior cord syndrome (Disruption of posterior spinal cord)

5) Brown-Sequard syndrome (Hemisection lesion of the spinal cord)

204
Q

What is complete spinal cord injury

A

Loss of motor and sensory function below the SCI level

E.g. high cervical cord level - quadriplegia, respiratory insufficiency, loss of bladder and bowel function, neurogenic shock

205
Q

What is Brown Sequand syndrome?

A
  • Hemisection lesion of the spinal cord, results in:
  • Unilateral spastic paralysis
  • Ipsilateral loss of vibration and proprioception (DCML - decussate at medulla oblongata)
  • Contralateral loss of pain and temperature sensation (spinothalamic tract - decussates in spinal cord)
206
Q

What investigations/tests are used to diagnose SCC?

A

Primary investigations:

  • Full neurological examination: tone, power, sensation, reflexes, proprioception
  • MRI of whole spine: diagnostic, and shows affected area as compressed. But spinal cord injury without radiographic abnormality (SCIWRA) may occur (cannot detect on MRI)
207
Q

What is the acute general management plan for SCC?

A

Depends on the underlying cause and requires MDT input.

1) VTE prophylaxis: due to reduced mobility

2) Nutritional input: if dysphagia or reduced oral intake

3) Blood pressure support

4) Ventilatory support: if high cervical lesion affects respiratory effort

5) Physiotherapy

208
Q

What is the management plan for acute traumatic spinal cord compression?

A

1) Immediate immobilisation

2) Definitive treatment: surgical spinal cord decompression and IV corticosteroids (not used in SCC caused by gunshots)

209
Q

What is the management plan for malignant spinal cord compression?

A
  • Dexamethasone: reduce inflammation
  • Radiotherapy: usually performed as a palliative measure in metastatic disease
  • Surgery: depending on impact of radiotherapy, age and prognosis
210
Q

What is the management plan for infective spinal cord compression (e.g. epidural abscess)?

A
  • IV antibiotics
  • Surgery: spinal cord decompression vs CT-guided needle aspiration if antibiotic therapy is insufficient
211
Q

Define sciatica

A

Sciatica refers to the symptoms associated with irritation of the sciatic nerve.

212
Q

What are the most common causes of sciatica?

A

The main causes of sciatica are lumbosacral nerve root compression by:

1) Herniated disc

2) Spondylolisthesis (anterior displacement of a vertebra out of line with the one below)

3) Spinal stenosis

213
Q

What is the pathophysiology of sciatica?

A

The spinal nerves L4 – S3 come together to form the sciatic nerve.

The sciatic nerve supplies sensation to the lateral lower leg and the foot. It supplies motor function to the posterior thigh, lower leg and foot.

Sciatica causes unilateral pain from the buttock radiating down the back of the thigh to below the knee or feet.

214
Q

What symptoms might a patient with sciatica present with?

A
  • Unilateral “electric” or “shooting” pain from buttock > back of thigh > below knee or feet
  • Other symptoms:
  • Paraesthesia (pins and needles)
  • Numbness
  • Motor weakness

Bilateral sciatica is a red flag for cauda equina syndrome.

215
Q

What investigations and tests are used to diagnose sciatica?

A
  • Sciatic stretch test
  • Patient lies on back, examiner lifts one leg to 80 - 90 degrees
  • Examiner dorsiflexes patient’s ankle
  • Pain buttock/posterior thigh indicates sciatic nerve root irritation
  • Symptoms improve with flexing the knee.
216
Q

What is the management plan for sciatica?

A

Low risk of chronic back pain:

  • Self-management, education, reassurance
  • Analgesia (1st line is NSAIDS, then codeine, benzodapines (diazepam))
  • Staying active and continuing to mobilise as tolerated

Additional options for medium/high risk:

  • Physiotherapy
  • Group exercise
  • Cognitive behavioural therapy

Specialist management options for chronic sciatica include:

  • Epidural corticosteroid injections
  • Local anaesthetic injections
  • Radiofrequency denervation - radiofrequency is used to target the nerves that supply sensation to the joints associated with back pain
  • Spinal decompression
217
Q

Define motor neuron disease

A

A group of neurodegenerative disorders characterised by the selective loss of neurons in the motor cortex, cranial nerve nuclei and anterior horn cells. Motor neurone disease is a progressive, ultimately fatal condition where the motor neurons stop functioning. There is no effect on the sensory neurons and patients should not experience any sensory symptoms.

Amyotrophic lateral sclerosis (ALS) is the most common (50% cases)

218
Q

What is ALS?

A
  • A condition characterised by the loss of neurons in the motor cortex and anterior horn
  • UMN (mainly corticospinal tract affected) + LMN signs (anterior horn cells affected)
  • Babinski response - indicates UMN lesion, non-specific (toe dorsiflexion on plantar stimulation)
  • Upper extremity weakness (UMN/LMN)
  • Stiffness, with poor co-ordination and balance (LMN)
  • Spastic, unsteady gait (UMN weakness affecting lower limbs)
  • The familial form is associated with mutations in superoxide dismutase (SOD1)
219
Q

What is progressive muscular atrophy?

A

LMN signs: predominantly affects the anterior horn cells
Best prognosis

220
Q

What is primary lateral sclerosis?

A

UMN signs: predominantly affects the corticospinal tracts - which transmit motor control of voluntary movements from the primary motor cortex to effector muscles.

221
Q

What is progressive bulbar palsy?

A

Affects the suprabulbar nuclei and cranial nerves, producing speech and swallow issues

  • Dysphagia, chewing difficulties
  • Flaccid tongue
  • Speech is hoarse, quiet, nasal

Worst prognosis

222
Q

What are the upper motor neurons?

A

UMN carry motor impulses from the brain to the LMN.

Neurotransmitter = glutamate

223
Q

What are the UMN signs of weakness (UMN lesions)?

A

Hypertonia - everything goes up

  • Spasticity (combo of paralysis, increased deep tendon reflexes and hypertonia)
  • Clonus ( involuntary muscle contractions)
  • Increased reflexes (hyperreflexia)

More detail:

  • Increased muscle tone→ Spasticity
  • Clasp-knife response: Initial higher resistance to movement is followed by a lesser resistance
  • Increase in deep tendon reflexes (e.g. biceps) and decrease in superficial reflexes (e.g. abdominal)
  • Flexors weaker than extensors in the legs
  • Extensors weaker than flexors in the arms
  • Babinski sign: Big toe raised rather than curled when the bottom of the foot is stimulated
  • Minimal atrophy and fasciculations
224
Q

What are the LMN signs of weakness (LMN lesions)?

A
  • Hypotonia - everything goes down
  • Muscle wasting (atrophy)
  • Reduced reflexes (hyporeflexia)
  • Fasciculations

More detail:
- Decreased muscle tone

  • Muscle paralysis
  • Weakness and wasting (atrophy)
  • Arreflexia (Absence of the relevant reflex)
  • Muscle fasciculation (twitching)- results from random and spontaneous depolarization of a motor neuron or axon
225
Q

What are the lower motor neurons?

A

LMN carry motor impulses from the UMN to muscles

Neurotransmitter = acetylecholine

226
Q

What are the risk factors for developing MND?

A
  • Advancing age = onset > 40 and average age 60
  • Male gender: 1.2 to 1.8x more likely
  • Family history: familial in 5% of cases and the SOD1 mutation is implicated
  • Smoking: ‘probable’ risk factor
227
Q

What investigations/tests are used to diagnose MND?

A

Clinical diagnosis, but investigations can be conducted to rule out other pathology

Investigations to consider:

  • Electromyography: in MND there will be evidence of fibrillation potentials (irregular contractions)
  • Nerve conduction studies: may show modest reductions in amplitude
  • MRI spine: imaging can help exclude spinal pathology which may mimic MND, such as cervical cord compression and myelopathy
  • Pulmonary function tests: patients with MND are at risk of respiratory failure
228
Q

What is the management plan for MND?

A

1) Riluzole: ALS - prolongs survival by 2-4 months, thought to protect motor neurons from glutamate-induced damage

2) Respiratory support: patients with reduced FVC (forced vital capacity) can use non-invasive ventilation at home, usually BiPAP; prolongs survival by 7 months

3) Supportive treatment:

  • Antispasmodics: such as baclofen
  • Feeding and nutritional support: often with PEG tube (directly into stomach)
  • Speech and language therapy
  • Physiotherapy
229
Q

What does CN III (oculomotor) innervate, what happens if there is a lesion on it?

A

Motor innervation to all extraocular muscles (SR, MR, IR, IO, LPS) apart from lateral rectus and superior oblique.

Dysfunction = ptosis, down and out eye and fixed dilated pupil

230
Q

What does CN IV (trochlear) innervate, and what happens if there is a lesion?

A

Superior oblique of the eye (intorsion, depression and abduction)

Dysfunction (caused by trauma) - diplopia (double vision) when looking down

231
Q

What does CNV innervate? What does a lesion on CN V cause?

A

Trigeminal - ophthalmic, maxillary and mandibular

Ophthalmic - sensation to frontal, ethmoid and sphenoid sinuses

Maxillary - sensation to maxillary and sphenoid sinuses

Mandibular - sensation to anterior 2/3 tongue
Motor to mastication muscles

Lesion causes:

  • Jaw deviates towards the affected side
  • Loss of corneal reflex
  • Trigeminal neuralgia
232
Q

What does CN VI normally innervate? What does a lesion on CN VI cause?

A

Innervates lateral rectus - abduction of eye

Ipsilateral loss of gaze abduction from midline

Adducted eye - a sign of increased intracranial pressure

233
Q

What does CN VII innervate? What does a lesion on CN VII cause?

A
  • Motor: Facial muscles
  • Sensory: taste to anterior 2/3 tongue

Parasympathetic:

  • Lacrimal glands
  • Submandibular & sublingual salivary glands

Lesions cause:

  • Facial drooping with NO forehead sparing
  • Bell’s palsy (facial paralysis with temporary inability to control the facial muscles on the affected side)

Treatment for Bell’s palsy = oral prednisolone

234
Q

What does CN VIII innervate? What does a lesion on CN VIII cause?

A

Vestibules and cochlea, invovled in:

  • Hearing (cochlear branch)
  • Balance (vestibular branch)

Lesions cause:

  • Hearing loss
  • Loss of balance
  • Caused by Paget’s disease, compression or middle ear disease
235
Q

What do CN IX (glossopharyngeal) and X (vagus) innervate? What do a lesion on CN IX and X cause?

A

CN IX:

  • Motor innervation to parts of the larynx
  • Taste and sensation to posterior third of tongue
  • Parasym to parotid glands

CN X:

  • Sensory innervation to oesophagus, lungs, trachea, heart and most of digestive tract, parts of the pharynx
  • Motor innervation to larynx
  • Parasym to trachea smooth muscles, bronchi, GI tract and regulates heart rhythm

Lesions will cause:

  • Impaired gag reflex
  • Swallowing, respiratory, GI and vocal difficulties, urinary retention
  • uvular deviation to the unaffected side
  • Caused by jugular foreman lesion
236
Q

What does a lesion on CN XI cause?

A

Innervates sternocleidomastoid (turn head side to side) & trapezius muscle (shrug shoulders) - so can’t turn head or shrug shoulders.

237
Q

What does a lesion on CN XII cause?

A

Motor innervation of the tongue normally, so lesion causes the tongue to deviate to side of lesion.

238
Q

Define Myasthenia Gravis

A

Myasthenia gravis is a chronic autoimmune disorder of the postsynaptic membrane at the neuromuscular junction of skeletal muscle.

It occurs due to circulating autoantibodies against the nicotinic acetylcholine receptor (AChR).

Some patients also have antibodies against muscle-specific kinase (MuSK), a postsynaptic protein.

239
Q

What are the risk factors for developing myasthenia gravis?

A
  • Female - 2x more likely
  • Family history of autoimmune disease
240
Q

What is the pathophysiology of Myasthenia Gravis?

A

Myasthenia Gravis occurs due to circulating autoantibodies against the nicotinic acetylcholine receptor (AChR). A small proportion of patients also have antibodies against muscle-specific kinase (MuSK), a postsynaptic protein.

Ultimately, this causes fewer available binding sites for acetylcholine (Ach) at the postsynaptic membrane, resulting in muscle weakness.

241
Q

What signs and symptoms might a patient with myasthenia gravis present with?

A

Hallmark symptom - fatigability - muscles become progressively weaker during activity and improve upon rest. Extraocular muscles are first affected, presenting with diplopia.

Symptoms

  • Lethargy
  • Muscle weakness gets worse at the end of the day
  • Double vision
  • Slurred speech
  • Dysphagia (swallowing difficulties)
  • Shortness of breath: may suggest a myasthenic crisis

Signs:

  • Proximal muscle weakness with fatigability: often affecting the face and neck
  • Ptosis exacerbated on upward gaze: may be bilateral or unilateral
  • Myasthenic snarl: a ‘snarling’ expression when attempting to smile
242
Q

What investigations/tests are used to diagnose myasthenia gravis?

A

Primary investigations:

1) Antibodies: AchR antibodies are 1st line, whilst anti-MuSK can also be tested in patients who are AchR antibody negative

2) Electrophysiological studies: repetitive nerve stimulation shows a decremental muscle response

3) CT thorax: to exclude a thymoma

4) Thyroid function tests: as there is a higher prevalence of autoimmune thyroiditis

243
Q

What physical examinations can elicit fatiguability in the muscle to test for myasthenia gravis?

A
  • Repeated blinking will exacerbate ptosis
  • Prolonged upward gazing will exacerbate diplopia on further eye movement testing
  • Repeated abduction of one arm 20 times will result in unilateral weakness when comparing both sides
244
Q

What is the treatment for a patient with stable myasthenia gravis?

A

1st line: acute treatment is prednisolone, long-term is pyridostigmine; an acetylcholinesterase inhibitor

245
Q

What is a myasthenic crisis?

A

A complication of myasthenia gravis due to the weakening of the respiratory muscles and is often provoked by infections or medications.

Patients present with increasing SOB > respiratory failure

246
Q

What is the management plan for a myasthenic crisis?

A
  • IV immunoglobulin or plasmapheresis (plasma exchange)
  • Non-invasive ventilation with BiPAP (Bilevel positive airway pressure)
  • Intubation: if severe respiratory compromise
  • Corticosteroids may be used as adjuvant therapy
247
Q

Define cauda equina syndrome (CES)

A

Cauda equina syndrome (CES) is a neurosurgical emergency which occurs when the bundle of nerves below the end of the spinal cord are compressed.

248
Q

What is the cauda equina?

A

The cauda equina, which is Latin for ‘horse’s tail’, is a bundle of lumbar, sacral and coccygeal nerve roots (L1-S5).

At around the vertebral level of L1, the spinal cord terminates as the conus medullaris, and the cauda equina begins. These nerves supply the pelvic organs and lower limbs:

Sensation to the lower limbs, perineum, bladder and rectum

Motor innervation to the lower limbs and the anal and urethral sphincters

Parasympathetic innervation of the bladder and rectum

249
Q

What is the aetiology of CES?

A

Lumbar disc herniation: the most common cause of CES

Trauma

Spinal tumour

Lumbar spinal stenosis: narrowing of the spinal cord may result in CES. Some causes include spinal osteoarthritis (spondylosis)

Epidural abscess or haematoma

250
Q

What signs and symptoms might a patient with CES present with?

A

Symptoms
1) Lower back pain and sciatica (97%)

2) RED FLAG Saddle anaesthesia’: numbness in the peri-anal region, groin and inner thighs (93%) - patients might describe this as ‘‘numbness when wiping”

3) Bladder and bowel dysfunction: urinary retention and faecal incontinence

4) Lower limb weakness

Signs

1) LMN signs (ipsilateral hypotonia, fasciculations and hyporeflexia)

2) Palpable bladder due to urinary retention

251
Q

What investigations/tests are used to diagnose CES?

A

Primary investigations:

MRI spine: gold-standard investigation and must be requested urgently - visualisation of lesion and compression of neural structures

Bladder ultrasound: to determine whether urinary retention

252
Q

What are the differential diagnoses for cauda equina syndrome?

A

Metastatic spinal cord compression (MSCC) - when a metastatic lesion compresses the spinal cord (before the end of the spinal cord and the start of the cauda equina).

MSCC presents similarly to cauda equina, with back pain and motor and sensory signs and symptoms.

A key feature is back pain that is worse on coughing or straining.

MSCC is an oncological emergency and requires rapid imaging and management.

253
Q

What is the management plan for cauda equina syndrome?

A

Emergency decompressive laminectomy (operation to remove vertebral bone (lamina): surgery should be performed within 24-48 hours of symptom onset

All patients with suspected CES should be urgently referred to neurosurgery.

Corticosteroids or radiotherapy: may be considered in certain patients with CES secondary to malignancy

254
Q

Atracurium

1) use
2) MOA
3) Side effects

A

1) Non-polarising neuromuscular blockade - muscle relaxant during surgery

2) Atracurium antagonizes the neurotransmitter action of acetylcholine by binding competitively with cholinergic receptor sites at the NMJ

3) Flushing; hypotension

255
Q

Diamorphine hydrochloride

1) Use
2) MOA
3) Side effects

A

1) Acute pain

2) It behaves as an agonist at a complex group of receptors (the μ (mu), κ (kappa) and δ (delta subtypes) that are normally acted upon by endorphins.

3) Arrhythmias; confusion; constipation; dizziness; drowsiness; eurphoria

256
Q

Ephedrine

1) Uses
2) MOA
3) Side effects

A

1) Reversal of hypotension from spinal or epidural anaesthesia, reversible airway obstruction

2) Increases BP by stimulating heart rate, and cardiac output. Causes bronchodilation due to the activation of beta-adrenergic receptors in the lungs.

3) Anxiety; headache; insomnia; nausea

257
Q

Paracetamol

1) Uses
2) MOA
3) Side effects

A

1) Pyrexia, mild to moderate pain

2) a weak inhibitor of prostaglandin synthesis, a molecule involved in pain and inflammation

3) Rare: thrombocytopenia, abnormal livre function

258
Q

Describe paracetamol overdose

A

Excessive ingestion (toxic overdose is defined as ingestion of paracetamol >75mg/kg) leading to acute liver failure

Normally, 90% conjugated to glucuronic acid in the liver > non-toxic compounds > excreted in urine

A small proportion is converted by cytochrome P450 to toxic metabolite > detoxified by conjugation with glutathione.

Glutathione depleted in overdose > hepatic necrosis

259
Q

What is the treatment for paracetamol overdose?

A

If ingestion less than 1 hour ago + dose >150mg/kg: Activated charcoal

If staggered overdose or ingestion >15 hours ago: N-acetylcysteine

If ingestion <4 hours ago: Wait until 4 hours to take a level and treat with N-acetylcysteine based on level

If ingestion 4-15 hours ago: Take an immediate level and treat based on level

260
Q

Suxamethonium

1) Use
2) MOA
3) Side effects

A

1) Anaesthesia as a skeletal muscle relaxant to facilitate tracheal intubation and mechanical ventilation

2) Neuromuscular blockade by mimicking acetylcholine at the NMJ, but hydrolysis is much slower, and depolarisation is prolonged

3) Arrhythmias; bradycardia (repeated doses); flushing

261
Q

Tramadol

1) Use
2) MOA
3) Side effect

A

1) Opioid painkiller for moderate to severe pain

2) Inhibits the reuptake of the natural neurotransmitters noradrenaline and serotonin and also binds weakly to μ-opioid receptors, blocking pain signal transmission to the brain

3) Arrhythmias; confusion; constipation

262
Q

Define syncope

A

Syncope refers to a transient loss of consciousness due to disruption of blood flow to the brain, often leading to a fall.

263
Q

Describe the 3 types of syncope and briefly explain what each is

A

1) Neurally-mediated reflex syncope -

  • Vasovagal syncope, carotid sinus syncope and situational faint
    Parasympathetic hyperactivity and/or sympathetic hypoactivity → ↓CO

2) Orthostatic syncope - failure of compensatory mechanisms to maintain CO when standing up

3) Cardiac syncope: arrhythmias or structural defects leading to reduced CO

264
Q

Give three examples of neurally-mediated syncope

A
  • Vasovagal - Emotional stress (primary)
  • Vasovagal - extended standing in a warm environment, such as a school assembly (primary)
  • Situational - Post-exercise (primary)
  • Carotid - carotid sinus massage
265
Q

Give three examples of orthostatic syncope.

A
  • Parkinson’s disease because of meds (primary)
  • Dehydration (primary)
  • Diabetes because of dehydration due to frequent urination and damage to blood vessels (secondary)
266
Q

Give three examples of cardiac syncope

A
  • AV node block (arrhythmia)
  • Valvular heart disease (structural)
  • Ischaemic heart disease (structural)
267
Q

What investigations/tests are used to diagnose syncope?

A

Depends on cause:

Lying and standing blood pressure: to look for underlying orthostatic hypotension

fall in SBP ≥ 20 mmHg and/or DBP ≥ 10 mmHg within 3 minutes of standing

ECG: assess for cardiogenic cause

FBC: to check for anaemia

Glucose: hypoglycaemia can present similarly
B-hCG: assess for ectopic pregnancy in females presenting with syncope

268
Q

What is the management plan/treatment for syncope

A

Neurally-mediated syncope:

  • Avoid triggers in primary syncope.

Management of underlying pathology:

Cardiac syncope

  • AV node block - packmaker indicated for high-grade

Orthostatic syncope

  • Medication review: anti-hypertensives, diuretics can cause orthostatic syncope
  • Hydration: ensure good fluid intake
  • Compression stocking: prevent venous pooling
269
Q

What are some DDx for syncope?

A

Acute coronary syndrome, ventricular arrhythmia, AV node block

270
Q

Define peripheral (diabetic) neuropathy

A

A condition that develops due to damage to extremities, e.g. hands, feet, and arms.

Diabetes mellitus type 1 and 2 are the most common cause of peripheral neuropathy in the UK - known as diabetic neuropathy.

Hyperglycaemia and hypertriglyceridemia contribute to the pathogenesis of neuropathy in T1 and T2 DM.

271
Q

What signs and symptoms might a patient with diabetic neuropathy present with?

A
  • Usually asymptomatic but can present with:
  • Pain (peripheral)
  • Loss of sensation (peripheral)
  • Dysaesthesia (peripheral) - (abnormal sensation of burning, tingling, and numbness due to peripheral nerve lesion)
  • Reduced or absent ankle reflexes (peripheral)
  • Painless injuries (peripheral)
272
Q

What investigations and tests are used to diagnose diabetic (peripheral) neuropathy?

A
  • Clinical diagnosis, but other routine tests can help rule out other causes
  • Fasting blood glucose - ≥ 7mmol/L - patients with peripheral neuropathy might not know they have DM
  • HbA1c > 6.5%/48mmol/mol - correlates with the level of glycaemic control
273
Q

What is the management plan for diabetic (peripheral) neuropathy?

A

Without pain:

  • Glycaemic control through diet, exercise and medication (e.g. insulin injections) can help relieve neuropathic symptoms

With pain

  • Glycaemic control as above and gabapentin as 1st line for painful DN
274
Q

DDx for diabetic (peripheral) neuropathy

A
  • Uraemia
  • Vitamin B12 deficiency
  • Hypothyroidism
275
Q

Define carpal tunnel syndrome.

A

Carpal tunnel syndrome is caused by compression of the medial nerve as it travels through the carpal tunnel in the wrist, causing pain and numbness in the median nerve distribution on the hand.

276
Q

What is the pathophysiology of carpal tunnel syndrome?

A

The flexor retinaculum is a fibrous band that wraps across the palmar side of the wrist.

Carpal tunnel runs through between carpal bones and the flexor retinaculum. It contains:

  • Median nerve
  • Forearm flexor tendons

The palmar digital cutaneous branch of the median nerve which runs through the carpal tunnel, supplies sensory innervation of the palms and thumb, index, middle and lateral half of ring finger

The median nerve = motor supply for the three thenar muscles = thumb movement

  • Abductor pollicis brevis (thumb abduction)
  • Opponens pollicis (thumb opposition – reaching across the palm to touch the tips of the fingers)
  • Flexor pollicis brevis (thumb flexion)

Therefore, the above sensory and motor innervations are affected in carpal tunnel syndrome aka compression of contents of the carpal tunnel

277
Q

What signs and symptoms might a patient with carpal tunnel syndrome present with?

A

Sensory symptoms in the areas supplied by palmar digital cutaneous branch of the median nerve - palms, thumb, index, middle and lateral half of ring finger:

  • Numbness
  • Paraesthesia (pins and needles or tingling)
  • Burning sensation
  • Pain

Worse at night, wake patient up, shake hands to relieve symptoms

Motor symptoms of carpal tunnel syndrome affect the thenar muscles, with:

  • Weakness of thumb movements
  • Weakness grip
  • Difficulty with fine movements involving the thumb
  • Thenar muscle atrophy
278
Q

What investigations/tests are used to diagnose carpal tunnel syndrome?

A
  • Phalen’s test - ask patient to flex wrist by putting back of two hands together - triggers sensory symptoms including numbness and paraesthesia
  • Tinel’s test - doctors tap patient’s wrist where the median nerve travels through the carpal tunnel - numbness and paraesthesia in the median nerve distribution
  • Electromyogram - gold standard test to see how well small electrical currents travel through median nerve
279
Q

What is the management plan for carpal tunnel syndrome?

A

Mild

Conservative - rest and altered activity

1st line: wrist splints (neutral or 20° extension) worn every night for 1 month.

Moderate/severe

1st line: surgical release

  • Open (with a vertical incision at the wrist) or endoscopic (keyhole) surgery. The flexor retinaculum (AKA transverse carpal ligament) is cut to release the pressure on the median nerve.
280
Q

What are the main DDx for carpal tunnel syndrome?

A
  • Osteoarthritis (OA)
  • Stroke, acute
281
Q

What is foot drop?

A

Difficulties with foot dorsiflexion due to peroneal neuropathy.

The common peroneal nerve It supplies the tibialis anterior muscle (via its branch the deep peroneal nerve).

This muscle dorsiflexes the ankle and therefore damage to its innervation leads to foot drop.

Most often caused by:

  • Sports injuries
  • Herniated spinal disc
  • Peripheral neuropathy caused by diabetes
  • Hip or knee replacement surgery
282
Q

Define pain (causes and management of pain)

A

An unpleasant sensory and emotional experience associated with actual or potential tissue damage.

283
Q

What are the 2 categories of pain?

A
  • Acute pain – new onset of pain
  • Chronic pain – pain present ≥ 3 months
284
Q

What are the two aspects of experiencing pain?

A

Pain is subjective

  • Sensory – the sensory signal transmitted from the nociceptors (pain receptor) (“it is a sharp sensation, likely a needle”)
  • Affective – the unpleasant emotional reaction to the pain (“it is excruciating, I can’t bear it”)
285
Q

How. is pain detected and processed by the body?

A

Pain receptors (nociceptors) at the ends of nerves detect damage/potential damage to tissues.

Nerve signals transmitted along the afferent sensory nerves to the spinal cord (primary afferent nociceptors). Part of PNS.

Two groups of nerve fibres transmit pain:

  • C fibres (unmyelinated and small diameter) – transmit signals slowly and produce dull and diffuse pain sensations
  • A-delta fibres (myelinated and larger diameter) – transmit signals fast and produce sharp and localised pain sensations

Signal travels to CNS through the spinal cord using the spinothalamic tract to the brain where it is interpreted as pain, mainly in the thalamus and cortex.

286
Q

Describe the analgesia ladder

A

There are three steps to the analgesic ladder: treat with lower steps before moving up if ineffective.

Step 1: Non-opioid medications such as paracetamol and NSAIDs

Step 2: Weak opioids such as codeine and tramadol (an SNRI and agonist of opioid receptors)

Step 3: Strong opioids such as morphine, oxycodone, fentanyl and buprenorphine

Other meds that can be combined with the above or used to treat neuropathic pain

  • Amitriptyline – a tricyclic antidepressant
  • Gabapentin – an anticonvulsant
  • Capsaicin cream (topical) – from chilli peppers
287
Q

Define brain tumours

A

Abnormal growths within the brain.

They vary from benign tumours (e.g. meningiomas) to highly malignant (e.g. glioblastomas).

Often asymptomatic when small and present with focal neurological symptoms and symptoms of raised ICP as they increase in size.

288
Q

What are gliomas?

A

Gliomas are tumours of the glial cells in the brain or spinal cord. There are three types from most to least malignant:

  • Astrocytoma - glioblastoma multiforme is the most common (95%)
  • Oligodendroglioma < 5%
  • Ependymoma
289
Q

What are meningiomas?

A

Tumours growing from the cells of the meninges in the brain and spinal cord.

They are usually benign, however they take up space and this mass effect can lead to raised intracranial pressure and neurological symptoms.

290
Q

What are pituitary tumours?

A

Benign tumours arising from the pituitary glands.

Can press on optic chiasm to cause bitemporal hemianopia.

They can also cause hormone deficiencies (hypopituitarism) or to release excessive hormones leading to:

  • Acromegaly
  • Hyperprolactinaemia
  • Cushing’s disease
  • Thyrotoxicosis
291
Q

Where might secondary brain tumours arise from?

A
  • Lung (NSCC).
  • Breast.
  • Malignant melanoma.
  • Kidney.
292
Q

What are the 3 cardinal presenting symptoms of brain tumours?

A
  1. Raised ICP.
  2. Progressive neurological deficit.
  3. Epilepsy
293
Q

Give 3 symptoms of raised ICP.

A
  1. Headache.
  2. Drowsiness.
  3. +/- vomiting.
294
Q

You ask a patient with a brain tumour about any factors that aggravate their headache. What might they say?

A
  1. Worst first thing in the morning.
  2. Worst when coughing, straining or bending forward.
295
Q

What is the cardinal physical sign of raised ICP?

A

Papilloedema - optic disc swelling

Due to raised intracranial pressure caused by obstruction of venous return from the retina.

296
Q

What re the other signs and symptoms of raised ICP?

A
  • Altered mental state
  • Visual field defects
  • Seizures (particularly focal)
  • Unilateral ptosis
  • Third and sixth nerve palsies
297
Q

Describe the WHO glioma grading.

A

Grade 1: astrocytoma - these are often removed with surgery and may not need further treatment.

Grade 2 astrocytoma, or a low-grade diffuse astrocytoma (may transform into high-grade tumour)

Grade 3: ‘anaplastic astrocytoma’ - cancer.

Grade 4: glioblastoma multiforme (GBM).

298
Q

What red flag symptoms in a headache should warrant further investigation?

A

Concerning features of a headache that should prompt further examination and investigation include:

  • Constant
  • Nocturnal
  • Worse on waking
  • Worse on coughing, straining or bending forward
  • Vomiting
299
Q

What investigations/tests are used to diagnose gliomas?

A
  • MRI head - diagnostic
  • Visual field testing - visual field defect may be present
  • CT head - used to rule out other causes of symptoms e.g. herniation, stroke, haemorrhage or when MRI unavailable
300
Q

What is the treatment/management plan for gliomas?

A

Initial management is dexamethasone and mannitol for cerebral oedema

Then:

  • Resective surgery.
  • Adjuvant chemotherapy with Temozolomide.
  • Palliative care.
301
Q

What is the management plan for secondary brain tumours?

A
  • Surgery and adjuvant radiotherapy.
  • Chemotherapy.
  • Supportive care.
302
Q

What is the management plan for pituitary tumours?

A
  • Trans-sphenoidal surgery
  • Radiotherapy
  • Bromocriptine to block prolactin-secreting tumours
  • Somatostatin analogues (e.g. octreotide) to block growth hormone-secreting tumours
303
Q

DDx for gliomas

A
  • Brain metastasis (secondary brain tumour)
  • Brain abscess
  • Multiple sclerosis
304
Q

Sodium valproate

1) Use
2) MOA
3) Side effects
4) Contraindications

A

1) Epilepsy, generalized seizures, partial seizures

2) sodium channel blocker - block voltage-gated Na+ channels; inhibition of action potentials in excitatory neurons; ↓ seizure activity

3) Gastrointestinal problems, thrombocytopenia, leukopenia, DRESS (allergic reaction - Drug reaction with eosinophilia and systemic symptoms)

4) Tetrageneic

305
Q

Carbamazepine

1) Use
2) MOA
3) Side effects
4) Containdications

A

1) Epilepsy, generalized seizures, partial seizures - not suitable for people with AV block and Hx of bone marrow depression

2) sodium channel blocker - block voltage-gated Na+ channels; inhibition of action potentials in excitatory neurons; ↓ seizure activity

3) Pancreatitis, hepatotoxicity, renal dysfunction, AV block

Boxed warning: SJS (Steven-Johnson syndrome), bone marrow depression

306
Q

DDx for ischaemic stroke

A
  • Intracerebral haemorrhage
  • TIA
  • Hypoglycaemia
307
Q

DDx for haemorrhagic stroke

A
  • Ischaemic stroke
  • Hypoglycaemia
308
Q

DDx for subdural haematoma

A
  • Epidural haematoma - “lucid” interval, associated with skull fracture
  • Intracerebral haematoma
  • Stroke
309
Q

DDx for subarachnoid haemorrhage

A

Aortic dissection

310
Q

DDx for bacterial meningitis

A
  • Encephalitis
  • Viral meningitis
  • Drug-induced meningitis

DDx for viral meningitis is bacterial meningitis and the other two above

311
Q

Generalised seizures: what is an generalised non-motor (absence) seizure?

A

A seizure that involves brief changes in awareness - typically involves behavioural arrest or staring lasting 5 to 10 seconds

312
Q

What are Acoustic Neuroma (AKA Vestibular Schwannoma)?

A

Acoustic neuromas are tumours of the Schwann cells surrounding the auditory nerve that innervates the inner ear.

They are slow-growing but eventually grow large enough to produce symptoms and become dangerous.

Acoustic neuromas are usually unilateral. Bilateral acoustic neuromas are associated with neurofibromatosis type 2.

Classic symptoms of an acoustic neuroma are:

Hearing loss
Tinnitus
Balance problems

They can also be associated with a facial nerve palsy.

313
Q

A 37 year old man presents to his GP with difficulty hearing in his left ear for six months. More recently, he is also experienced the sensation of the world spinning around him. On examination, he has reduced sensation of the ophthalmic, maxillary, and mandibular divisions of the left trigeminal nerve, reduced taste sensation of the posterior left side of the tongue; and deviation of the uvula to the right. The rest of his neurological examination is normal, with no other motor, sensory, or cerebellar deficit. What is the most likely diagnosis?

A

Acoustic neuroma, classically presenting with unilateral deafness and vertigo occurring later, in addition to left-sided lesions of cranial nerves V, IX and X